FINA 4325 Final tito

Ace your homework & exams now with Quizwiz!

If a retiree's annual Social Security benefit is $9,000 at age 62 and $9,600 at age 63, how many years would it take to equal the benefit of forgoing a year of payments (waiting one more year to receive a larger payment), not including inflation? A) 13 years B) 15 years C) 11 years D) 8 years

$9,000 divided by the extra amount ($600) equals 15 years to equal the same benefit taking the larger amount the next year. ($9,000 ÷ $600 = 15 years) LO 7.2.1

Which of the following types of plans is typically used by religious, charitable, educational, and other Section 501(c)(3) entities or public school systems? A) Section 403(b) plan/TSA B) SEP plan C) SIMPLE D) Section 457 plan

A Section 403(b) plan/TSA (tax-sheltered annuity) is a tax-deferred employee retirement plan that is adopted only by certain tax-exempt organizations and certain public schools and colleges. Employees have individual accounts to which employers contribute (or employees contribute through salary reductions).

Which of the following reasons for an early distribution from an IRA is NOT an exception to the 10% penalty? A) The plan owner becomes totally and permanently disabled B) Made on or after the account owner attains age 59½ C) A distribution made after age 55 and separation from service with an employer D) Early distributions made for qualifying medical expenses exceeding 10% of the account owner's AGI

A distribution made after age 55 and separation from service with an employer is not a qualified retirement plan early distribution penalty exception. LO 5.3.1

A fully insured Section 412(e)(3) pension plan is funded exclusively by A) blue-chip stocks. B) Treasury bonds. C) cash value life insurance or annuity contracts. D) municipal bonds.

A fully insured 412(e)(3) pension plan is funded exclusively by cash value life insurance or annuity contracts. Using insurance as a funding vehicle ensures the payment of a death benefit to plan beneficiaries. LO 8.2.1

If the client's business objectives are to reduce income tax, reward executive employees, retain and recruit employees, and reduce employee turnover, which plan selection approach could address these issues? A) Pension plan only approach B) Profit-sharing only approach C) None of these D) A pension plan or a profit-sharing plan

A pension plan or a profit-sharing plan can address each of these objectives. LO 8.1.1

Dave is covered by a qualified retirement plan. If the plan provides life insurance on Dave's life, which of the following statements regarding the income tax ramifications to Dave is CORRECT? Dave must include the pure protection cost of the life insurance in his income. The pure protection cost of the life insurance will be treated as nontaxable basis once Dave begins receiving distributions from the plan.

A qualified plan participant must include the pure protection cost of the life insurance as income, but this amount is treated as nontaxable basis once the participant begins receiving distributions from the plan. LO 8.2.2

Which of the following best describes the purpose of establishing a stretch IRA? A) To enable the owner to take distributions before age 59½ without paying the 10% penalty B) To allow the owner to borrow money from the IRA without committing a prohibited transaction C) To extend the period of tax-deferred earnings beyond the original owner's lifetime D) To allow the owner to continue making contributions to a traditional IRA beyond age 72

A stretch IRA is used to stretch the period of tax-deferred earnings on the IRA beyond the lifetime of the original owner. The goal is to delay the distribution of assets from the IRA for as long as possible. LO 5.2.1

Assume Bernard, age 55, is planning to retire in 10 years at age 65. He is a sole proprietor of a business with 15 employees but has not yet implemented a formal retirement plan for the business. Bernard's company currently has a strong cash flow which is expected to continue. His own personal savings retirement need is $85,000 per year, and Bernard pays himself only $95,000. The company can afford to contribute $100,000 this year for Bernard's account to any retirement plan that is implemented. Furthermore, Bernard will commit to an annual contribution necessary to fund the retirement plan if needed. Considering only this limited information, which of the following types of qualified retirement plans would you recommend for Bernard and his business? A) Stock bonus plan B) Money purchase pension plan C) Profit-sharing plan D) Traditional defined benefit pension plan

A traditional defined benefit pension plan is most appropriate for Bernard and his business. The business has favorable cash flow and can commit to the annual contribution required by the defined benefit approach. In addition, Bernard's savings need as a percentage of his compensation exceeds anything possible in a defined contribution plan. Finally, Bernard is currently age 55 with only 10 years until retirement. LO 8.1.2

Assume that a worker's Social Security full retirement age (FRA) is 67, and the worker retires and starts drawing Social Security early at age 64. What are the consequences? A) The worker receives a reduced benefit starting at age 64 but will receive the full benefit starting at age 67. B) The worker receives the full benefit at age 64 but receives a reduced benefit starting at age 70. C) The worker receives a reduced benefit for the rest of his life. D) The worker's retirement benefit will not receive cost-of-living increases until the year the person reaches FRA.

A worker who starts Social Security retirement benefits early (as early as age 62 is allowed) will receive a reduced benefit, which will continue for life. This reduced benefit will be adjusted for inflation each year. LO 7.1.1

Barry is the sole shareholder of Zippy Internet Services, Inc., a C corporation that employs 10 people. The company has been in business for four years and has had fluctuating cash flows during that time. Barry would like to install a qualified plan with the following criteria: Reward, motivate, and retain employees Reduce corporate income taxes Provide for his own retirement Barry would like to contribute to each employee's account an amount equal to 8% of compensation. What would be the most appropriate course of action for Barry? A) Establish a profit-sharing plan. B) Establish a defined benefit pension plan combined with a profit-sharing plan. C) Establish a traditional defined benefit pension plan. D) Establish a money purchase pension plan.

A. profit-sharing plan would accomplish all of Barry's objectives and is appropriate for a business with unstable cash flow. Defined benefit pension plans and money purchase pension plans require annual mandatory funding, making them inappropriate for businesses with unstable cash flows. LO 8.1.2

Jason has determined he will have an annual retirement income deficit. The deficit for the first year of retirement, 10 years from now, is $90,000. He expects to be in retirement for 30 years, and believes he can earn a 7% after-tax annual return on invested dollars. Inflation is expected to average 4% annually over this same period. What is the amount of lump-sum retirement funds needed by Jason at the beginning of retirement to fund his additional retirement income needs? (Round to the nearest dollar.) A) $1,842,297 B) $1,392,409 C) $1,790,644 D) $1,816,961

A. The lump-sum retirement fund needed at the beginning of retirement is $1,842,297, calculated as follows: In BEGIN mode: PMT = −$90,000 n = 30 i = 2.8846, or [(1.07 ÷ 1.04) − 1] × 100 PVAD = $1,842,297 LO 8.3.2

Which of the following is the most prudent approach for projecting a client's life expectancy during retirement? A) Disregard the client's own health status and family health history. B) Add a few years to provide a cushion against the client living longer than expected. C) Subtract a few years on the assumption that investment returns will be higher than expected. D) Rely only on actuarial life expectancy tables.

Actuarial life expectancy tables have some value in projecting a client's life expectancy, but they fail to take into account the client's own health status or family history. The most prudent approach is to add 5 to 10 years to the actuarial life expectancy to protect against the possibility the client will live longer than expected. LO 8.3.1

Which of the following statements regarding Section 403(b) plans is CORRECT? Section 403(b) plans must comply with many of the same reporting and auditing requirements that apply to Section 401(k) plans. Certain eligible participants in a Section 403(b) plan may defer as much as much as $29,000 into the plan in 2020. Section 403(b) plans may provide for plan loans to participants. Funding for Section 403(b) plans is limited to mutual funds and annuities.

All of the statements are correct. The $29,000 maximum elective deferral for 2020 includes the $19,500 basic limit; the extra $3,000 per year for up to $15,000 lifetime for employees of not-for-profit health care, education, and church employers; and the $6,500 catch-up for those age 50 and older.

Increases in which of the following can push a retiree's rate of inflation higher than the overall annual rate? Housing costs Travel goals Food prices Personal buying habits

All of the statements are correct. The rate of inflation for retiree-senior citizens may well be higher than a 3% per year average annual rate. This is primarily because of the rising cost of health care in the United States. In addition, a retiree's actual rate of inflation may vary significantly from the CPI figure because of regional housing or food prices, personal buying habits, and travel goals. LO 8.3.1

Karen, age 51, wishes to take distributions from her traditional IRA and avoid imposition of the 10% early distribution penalty. Which of the following distributions will allow Karen to avoid the penalty? Karen is totally and permanently disabled. Karen wants the distribution to pay medical expenses exceeding 10% of her adjusted gross income (AGI). Karen may take distributions under the substantially equal payments rule. Karen needs cash to pay for tuition for her child at State University.

All of these statements are correct. LO 6.3.1

Which of the following statements is(are) CORRECT regarding tax-free rollovers of qualified plan distributions pursuant a qualified domestic order (QDRO)? The recipient of a distribution from a qualified plan pursuant a QDRO may execute a tax-free rollover into another qualified plan. The recipient of a distribution from a qualified plan pursuant a QDRO may execute a tax-free rollover into a SEP. The recipient of a distribution from a qualified plan pursuant a QDRO may execute a tax-free rollover into a Section 403(b) plan. The recipient of a distribution from a qualified plan pursuant a QDRO may execute a tax-free rollover into a Section 457 plan.

All of these statements are correct. LO 6.5.1

A stretch IRA extends or stretches the period of tax-deferred earnings within an IRA, possibly over several generations. extends or stretches the period of tax-deferred earnings within an IRA beyond the lifetime of the original owner. allows the IRA owner's beneficiary to name his own beneficiary upon the owner's death.

All the statements are correct. LO 6.4.1

Myra, age 35, converted an $80,000 traditional IRA to a Roth IRA last year. Her adjusted basis in the traditional IRA is $20,000. She also makes a contribution of $5,000 to the same Roth IRA last year. Myra is in a combined 30% marginal tax rate. If Myra takes a $4,000 distribution from her Roth IRA this year, 2020, how much total federal tax, including penalties, is due as a result of the distribution? A) $1,200 B) $400 C) $1,600 D) $0

Although the distribution is not a qualified distribution, it will not be taxable income because it is treated as a distribution from the Roth IRA regular contributions first. Because the $4,000 distribution is not includible in gross income, nor does it relate to a conversion within the last five years, the distribution is not subject to regular income tax or the 10% early withdrawal penalty. Answer is D. LO 5.4.2

Stewart and Abby, both age 35, plan to contribute a total of $12,000 to their IRAs for this tax year. They both work outside the home, and they file a joint income tax return. Stewart is a teacher at the local high school and participates in a 403(b) plan. Abby's employer does not provide a retirement plan. They expect that their MAGI in 2020 will be $129,000. What amount, if any, can they deduct for their IRA contributions? A) $0 B) $12,000 C) $6,000 D) $7,000

An individual is not denied a deduction for his IRA contribution simply because of the other spouse's active participation, unless the couple's combined AGI exceeds $206,000 (2020). Based on their AGI, Abby will be able to deduct a contribution of up to $6,000 to an IRA. Since their combined AGI is too high for Stewart to make a deductible IRA contribution, he should consider contributing to a Roth IRA.

Which of the following statements regarding the basic provisions of Section 403(b) plans is CORRECT? A special catch-up provision is available to employees of Section 501(c)(3) organization employers who have at least 1 year of service. An eligible employee may be able to use both a special catch-up provision and an over-age-50 catch-up provision in the same year. TSAs are available to all eligible employees of Section 501(c)(3) organizations who adopt such a plan. If an employee has at least 15 years of service with an eligible employer, an additional catch-up contribution may be permitted.

Anser 2, 3, and 4 Statement I is incorrect. The special or additional catch-up provision requires at least 15 service years.

Which of the following would merit assuming an after-tax rate of return when considered as part of the accumulation strategy in a client's retirement planning calculations? Corporate zero coupon bonds held outside a qualified plan or IRA High dividend paying blue-chip stocks held in a Roth IRA Aggressive growth mutual funds held in a Section 457 plan sponsored for employees by a tax-exempt organization Cash value of a variable life insurance policy that is a modified endowment contract (MEC)

Answer 1 Only the corporate zero coupon bonds would generate currently taxable income which would merit assuming an after-tax rate of return in the accumulation strategy. The Roth IRA would generate tax-deferred earnings, at worst, but possibly tax-free earnings. The Section 457 plan offers tax-deferred earnings. The cash value of a variable life insurance policy that is a MEC has tax-deferred earnings until distributed as a withdrawal or policy loan. LO 8.2.1

The employer bears the investment risk for which of the following retirement plans? Cash balance pension plan SIMPLE 401(k) plan Age-weighted profit-sharing plan Money purchase pension plan

Answer 1 The employer bears the investment risk for the cash balance pension plan. For the remaining plans, the employee bears the investment risk. LO 8.2.1

Which of the following is considered an active participant for determining the deductibility of traditional IRA contributions this year? A participant in a defined benefit pension plan who has just satisfied the eligibility requirements and entered the plan in the past six months A participant in a traditional Section 401(k) plan who is currently not making elective deferrals but has $100 of forfeitures reallocated to her account this year A highly compensated employee with a $500,000 account balance in a profit-sharing plan for which the plan earnings this year are $35,000 but no employer contributions, employee contributions, or reallocated forfeitures were added this year A self-employed professional with no employees maintaining a simplified employee pension (SEP) with a $10,000 account balance funded by a 20% contribution two years ago plus earnings

Answer 1 and 2 Active participation for purposes of determining deductibility of IRA contributions differs from being covered under a qualified plan. Specifically, an employee must be contributing to the plan, having employer contributions or forfeitures reallocated on his behalf, or accruing a defined benefit before he is considered an active participant in a qualified plan for IRA purposes. The participant in Statement I has begun to accrue a benefit in the pension plan. The participant in Statement II is considered an active participant because of the forfeiture relocation. The taxpayers in Statements III and IV are not considered active participants this year because no contributions were made on their behalf this year. O 5.1.1

Maria has a traditional IRA valued at $500,000. She named her daughter, Faith, as beneficiary of the account. If Maria dies prematurely, which of the following statements is CORRECT? Faith inherits the IRA. Faith can transfer the inherited funds to an inherited IRA via a direct trustee-to-trustee transfer and name her own beneficiary. Because Faith is a nonspouse beneficiary, she is not allowed to roll over the IRA. Faith can roll over the IRA into her own Section 401(k) plan.

Answer 1 and 2 Statements I and II are correct. A nonspouse beneficiary (such as an adult child) may use a trustee-to-trustee transfer of the decedent's balance from a qualified plan, Section 403(b) plan, governmental Section 457 plan, or IRA to her own inherited IRA. However, the nonspouse beneficiary must generally begin receiving the distributions from the deceased participant's IRA immediately, whereas a surviving spouse beneficiary may continue to defer payouts until she attains age 72. LO 5.2.1

Janice is the self-employed owner of an unincorporated business. She has hired the following family members to work in her business. Which family member(s) is(are) covered by Social Security? Her husband Her daughter, age 16 Her son, age 19

Answer 1 and 3 A child under the age of 18 who is employed in the parent's unincorporated business is not covered by Social Security. LO 7.2.2

Which of the following statements regarding TSAs/Section 403(b) plans is CORRECT? The sponsor must be a tax-exempt organization that meets the requirements of Section 501(c)(3), a governmental organization, or public educational organization. Lump-sum distributions may be eligible for special long-term capital gain treatment. In-service withdrawals may be permitted. The plan can invest in individual stocks and bonds, but not options or futures.

Answer 1 and 3 Statement II is incorrect because distributions from a TSA are treated as ordinary income. Statement IV is incorrect. Only annuities and mutual funds are permitted investments in a TSA.

Which of the following statements regarding TSAs and Section 457 plans is CORRECT? Both plans may be funded entirely by participant contributions. Participation in either a TSA or a Section 457 plan will cause an individual to be considered an active participant for purposes of phasing out the deductibility of traditional IRA contributions. Both plans allow net unrealized appreciation tax treatment for lump-sum distributions. Both plans must meet minimum distribution requirements that apply to qualified plans.

Answer 1 and 4 Statements I and IV are correct. Statement II is incorrect because a Section 457 plan is a deferred compensation arrangement that will not cause a participant to be considered an active participant. Statement III is incorrect because net unrealized appreciation (NUA) tax treatment is not permitted for distributions from either plan.

Charles (age 38) has just died. He has been credited with the last 30 consecutive credits of Social Security coverage in the last 30 quarters since he left school and began full-time employment. He had never worked before leaving school. Which of the following persons are eligible to receive Social Security survivor benefits as a result of Charles's death? Charles's child, Bill, age 16 Charles's child, Dawn, age 19 Charles's widow, Maggie, age 38 Charles's dependent mother, Betty, age 60

Answer 1 only Dawn is too old under the rules, Maggie does not have a child under age 16 for whom she is caring, and Betty is not eligible because she is not age 62 or older. Only Bill is eligible to receive a dependent or surviving child's benefit. LO 7.2.1

Whose Social Security benefit is included in the calculation of the maximum family benefit for those eligible for benefits based on a retired worker's fully insured status? The retired worker's benefit The retired worker's former spouse A dependent child's benefit A caregiver spouse's benefit for caring for a qualified disabled child

Answer 1, 2 and 4 Only Statement II is incorrect. Benefits paid to a former spouse of a covered worker are not considered in the application of the family maximum benefit limit. LO 7.2.2

How does simplified employee pension (SEP) plan participation affect an employee's IRA contributions? The deductibility of an active participant's IRA contribution depends upon his MAGI. SEP plan participation does not reduce or eliminate an employee's ability to fund an IRA. Employees who participate in a SEP plan are considered active participants in an employer-sponsored retirement plan for the tax year in which an employer contribution is made. Employees who participate in a SEP plan are not considered active participants in an employer retirement plan for the tax year in which an employer contribution is made.

Answer 1, 2, 3 Statements I, II, and III are correct. Employees who participate in a SEP plan are considered active participants in an employer retirement plan for the tax year in which an employer contribution is made. The deductibility of an active participant's IRA contribution depends upon his MAGI and can be phased out or eliminated at certain income levels. SEP plan participation does not reduce or eliminate an employee's ability to fund an IRA. The IRA can be funded, but not necessarily deducted from gross income.

Which of the following statements regarding the incidental death benefit test for life insurance inclusion in a qualified plan is CORRECT? For defined contribution plans, no more than 25% of the pension plan's assets may be in the form of universal life insurance cash value. For defined benefit pension plans, the life insurance death benefit cannot exceed 100 times the expected monthly benefit for an employee.

Answer 2 For defined contribution plans, no more than 25% of the employer contributions (not plan assets) can be used to purchase universal life or term insurance. No more than 50% of the employer contributions in a defined contribution plan can be used to purchase whole life insurance. LO 8.2.2

Which of the following types of retirement plans would be suitable for a businessowner who is uncomfortable with the idea of mandatory annual contributions? Defined Benefit Pension Plan Profit-sharing plan

Answer 2 All defined benefit plans require mandatory annual funding. A defined benefit plan would not be appropriate for a businessowner who is uncomfortable with mandatory annual funding. LO 8.1.2

Which of the following are permitted investments in tax-sheltered annuity (TSA) plans? Individual stocks Group or individual annuity contracts Custodial accounts invested in mutual funds

Answer 2 and 3 Investments in TSAs are limited to annuity contracts, either group or individual, and custodial accounts invested in mutual funds. Individual stocks and bonds are not permitted.

Which of the following is a permitted investment in tax-sheltered annuity (TSA) plans? Individual stocks Group or individual annuity contracts Custodial accounts invested in mutual funds

Answer 2 and 3 Investments in TSAs are limited to annuity contracts, either group or individual, and custodial accounts invested in mutual funds. Individual stocks and bonds are not permitted. LO 8.2.1

Which of the following statements are disadvantages for the employer-sponsor of a cash balance pension plan? A certain level of plan benefit is guaranteed by the PBGC. The employer bears the investment risk in the plan. Cash balance pension plans are less expensive for the employer than a traditional defined benefit pension plan. Retirement benefits may be inadequate for older plan entrants.

Answer 2 and 4 A cash balance pension plan has several advantages and disadvantages. Advantages include the following: A certain level of plan benefits is guaranteed by the PBGC. There are significant cost savings for the employer as compared to the traditional defined benefit pension plan. Disadvantages include the following: The employer bears the risk of poor investment performance. The retirement benefits may be inadequate for older plan entrants. If the plan is a converted traditional defined benefit pension plan, the lump-sum payout at the employee's retirement date may be considerably less under the cash balance formula. This is a disadvantage to workers and an advantage to employers. LO 8.1.1

Martha has inherited a traditional IRA that contained no after-tax contributions. She would rather not take the required minimum distributions but instead roll the distributions over into her own IRA to save for her own retirement and avoid paying income tax. Which of the following statements is CORRECT? Martha should direct the IRA trustee to make an annual direct transfer to her own traditional IRA of the required minimum distributions so the distributions remain nontaxable. To minimize current taxation, Martha should execute a direct transfer of the entire IRA into an inherited IRA.

Answer 2 only Only Statement II is correct. Required minimum distributions may not be rolled over. To decrease current taxation, Martha should execute a direct transfer to an inherited IRA. She, will, however, be required to begin required minimum distributions from the inherited IRA. LO 5.2.1

Alice died five years ago. Joe, age 46, Alice's surviving spouse, has been raising their three children (John, Mary, and Susan). Joe earns over $95,000 a year and receives $900 per month from Social Security. Alice was insured under Social Security when she died. Which of the following relatives will be eligible for a survivor benefit this year? John, age 20, qualifies for the monthly child benefit because he is a full-time college student. Mary, who will be 19 in September, will continue to qualify until she graduates from high school in June. As the surviving spouse, Joe qualifies for the father's benefit while he is caring for a child under age 16, but the benefit is reduced to zero because of his earnings. Susan, who is age 15, qualifies for the child's monthly benefit.

Answer 2, 3, and 4 To qualify for the child's benefit, the child must be under age 18, or a full-time student at an elementary or secondary school if under age 19. A child may also qualify if the child became disabled before reaching age 22. Joe qualifies for the father's benefit, but that benefit is reduced by $1 for each $2 of earned income in excess of the limit for persons under full retirement age (FRA). Joe's income would reduce the benefit to zero. At John's age, he would only qualify for a benefit if he were disabled. LO 7.2.1

Which of the following statements most accurately describes the tax treatment of contributions to and distributions from a Roth IRA? Contributions are made with pretax dollars. A withdrawal from the account will not be subject to tax if the account has been established for at least three years and the funds (up to $10,000) are being used for a first-time home purchase. Distributions are not taxable if they are attributable to disability and the account has been established for at least five years. If the account has been open for at least five years and the account owner is age 59½, distributions are penalty free and income tax free.

Answer 3 and 4 Contributions to a Roth IRA are made with after-tax dollars. Distributions from a Roth IRA are income tax and penalty free if the owner has maintained the account for at least five years and the distribution is attributed to one of the following: Death Disability First-time home purchase ($10,000 lifetime maximum) Attainment of age 59½ LO 5.4.2

Which of the following is subject to the required minimum distribution (RMD) requirements after the account owner/plan participant dies? Traditional IRAs Roth IRAs Qualified plans

Answer All of them All of these retirement accounts are subject to RMD requirements after the account owner/plan participant dies. However, RMD requirements do not apply to Roth IRAs while the owner is alive. LO 5.4.1

The employer portion of the payroll tax rate, including the portion dedicated to Social Security (OASDI) and the portion dedicated to Medicare funding, on a covered worker's earnings up to the Social Security taxable wage base is A) 6.2%. B) 7.65%. C) 15.3%. D) 1.45%.

Answer B. Employers must pay a combined Social Security (OASDI) and Medicare rate of 7.65%. Employers pay 6.2% for Social Security and 1.45% for Medicare. The Social Security portion is payable up to the Social Security wage base of $137,700 for 2020. The Medicare portion is paid without an income limit. LO 7.1.1

Using the Uniform Lifetime Table to calculate the required minimum distributions (RMDs) from a qualified plan is mandatory unless A) there is more than 1 designated beneficiary. B) the designated beneficiary is the participant's spouse and the spouse is more than 10 years younger than the participant. C) the designated beneficiary is a child under the age of 16. D) there is no designated beneficiary

Answer B. The Uniform Lifetime Table must be used to calculate required minimum distributions (RMDs) under a qualified plan or IRA unless the designated beneficiary is the participant's spouse and the spouse is more than 10 years younger than the participant. LO 6.1.2

Marian, age 62, converts $30,000 from a traditional IRA to a Roth IRA in 2014. In 2016, she converts another traditional IRA with a fair market value of $35,000 to a Roth IRA. She makes no other IRA contributions. In 2020, Marian takes a $40,000 distribution from her Roth IRA. This distribution is treated as $30,000 from the 2014 conversion contribution and $10,000 from the 2016 conversion contribution, both of which were includable in her gross income when converted. As a result, for 2020, A) the $10,000 from the 2016 conversion is subject to the 10% penalty tax. B) the $10,000 withdrawal from the 2016 conversion is not subject to the 10% penalty tax. C) $10,000 is includable in Marian's gross income. D) $20,000 is includable in Marian's gross income.

Answer B. The conversion amounts were already included in Marian's gross income when converted. Therefore, they will not be subject to income taxes again when withdrawn. The distribution allocable to the $10,000 conversion contribution made in 2016 (less than five taxable years ago) starts out as being subject to the early distribution penalty because it was withdrawn less than five years after the conversion. However, it is not subject to the 10% penalty tax under Section 72(t) in this case because Marian is over age 59½, which is one of the exceptions to the 10% penalty. The withdrawal of the conversion amount from the first conversion is not subject to the 10% early withdrawal penalty because the conversion is more than five years old. Thus, if the owner would have been younger than age 59½, the withdrawal of that money would not have been subject to the early withdrawal penalty. LO 5.4.2

Which of the following statements is CORRECT? In a single-participant defined benefit pension plan, the employer-participant is assuming all of the investment risk. An owner-employer may not wish to assume the investment risk for employees

Answer Both 1 and 2 LO 8.1.1

Maryellen is considering naming her estate as the beneficiary of her traditional IRA. Which of the following is(are) a disadvantage of this approach? Her estate cannot be treated as a designated beneficiary for purposes of determining the distribution period after she dies. The estate will probably pay more income tax on the IRA distributions than would an individual beneficiary.

Answer both These are both potential disadvantages of naming one's estate as beneficiary. The estate cannot be treated as a designated beneficiary for purposes of determining required minimum distributions, and the estate will begin paying income tax at the maximum rate at a much lower level than an individual beneficiary would. LO 6.4.1

Jill and Mark are celebrating their 20th wedding anniversary, receiving 20-year watches from their employers, and reaching full retirement age (FRA), all on the same day. Which of the following statements correctly describe the Social Security benefit Jill is eligible to receive? A) 100% of her own benefit B) 75% of Mark's benefit C) 85% of her own benefit D) 100% of Mark's benefit

Answer is A. Currently, the spouse of a Social Security recipient is entitled to 50% of the recipient's primary insurance amount (PIA), subject to a family maximum, as long as the spouse is of full retirement age (FRA) or between age 62 and FRA for reduced benefits. The spouse will take the greater of either 100% of their own benefit or 50% of their spouse's benefit. LO 7.2.1

Which of the following constitutes an exception to the imposition of the 10% premature distribution penalty for distributions made from an IRA owned by an individual who is currently age 40? A) A distribution in payment of qualified higher education expenses B) A distribution paid as a series of substantially equal payments over a five-year period C) A distribution following separation from service at age 56 D) A distribution taken after demonstration of a financial hardship

Answer is A. IRA distributions are exempt from the early distribution penalty if made in payment of qualified higher education expenses. The other distributions are either exempt only if made from a qualified plan or not exempt, regardless of the source. The exception for a distribution paid as a series of substantially equal payments must extend over the greater of five years or age 59½. LO 5.3.1

Which of the following groups would NOT benefit from using Roth IRAs? A) Low-income wage earners who need current deductions B) Low-income wage earners who are active participants in their employers' plans and are not eligible to make deductible IRA contributions C) Taxpayers who anticipate being in a higher tax bracket in their retirement years D) High-income wage earners who have exhausted the tax benefits of other tax-favored vehicles

Answer is A. Roth IRAs offer no current deductions. Low-income wage earners needing current deductions are better served by traditional IRAs. LO 5.4.1

Mary is 66 years old and receives full old-age benefits from Social Security—in her case, $1,200 per month. Her husband, Ralph, age 67, who has not worked enough quarters outside the home to be covered in his own right, receives 50% of what Mary receives each month ($600). Assume that Mary dies tomorrow. What will Ralph's Social Security benefit be? A) $1,200 B) $600 + $1,200, or $1,800 C) $600

Answer is A. The $600 spousal benefit stops, and Ralph will begin receiving 100% of Mary's old-age Social Security benefit. This survivor benefit is not reduced because Ralph has reached his full retirement age (FRA) when his wife died. LO 7.2.1

Which of the following statements regarding the net unrealized appreciation (NUA) portion of employer stock received in a lump-sum distribution is CORRECT? The NUA portion is A) taxed at the capital gains rate when the stock is sold. B) taxed as ordinary income when the stock is sold. C) received tax free. D) taxed as ordinary income in the year of the distribution.

Answer is A. The NUA portion of the distribution is taxed at the capital gains rate when the stock is sold. The adjusted basis of the stock to the qualified plan trust is taxed as ordinary income to the participant in the year of the distribution.

Which of the following statements regarding the net unrealized appreciation (NUA) portion of employer stock received in a lump-sum distribution by a plan participant is CORRECT? A) The NUA portion of the stock value is taxed at the capital gains rate when the stock is sold. B) The NUA on the employer stock is taxed as ordinary income in the year of the distribution. C) When the taxpayer receives the employer stock, the stock is taxed as ordinary income when sold. D) The portion of the fair market value on the date of distribution that is NUA is tax free to the plan participant.

Answer is A. The adjusted basis of the stock to the qualified plan trust is taxed as ordinary income to the participant in the year of the distribution. LO 6.1.2

Jackie turns 70 years old on March 15, 2020. When must she begin taking distributions from her Section 401(k) plan, assuming she no longer works for the employer-sponsor of the plan? A) December 31, 2021 B) April 1, 2021 C) December 31, 2020 D) April 1, 2020

Answer is B. Generally, the first required minimum distribution (RMD) from a qualified plan must be taken by April 1 of the year following the year in which the participant reaches age 70½. Jackie reaches age 70½ on September 15, 2020, so she must begin taking distributions by April 1, 2021. Even though all or part of the first RMD distribution could be taken in 2021, it is still the 2020 RMD (until April 2, 2021). Also, the 2020 RMD will be based on the account balance at the end of 2019. LO 6.2.2

Which of the following statements regarding the basic provisions of tax-sheltered annuities (TSAs)/Section 403(b) plans is NOT correct? A) The special catch-up provision for eligible Section 403(b) participants allows up to a $39,000 (2 × $19,500 in 2020) elective deferral in the last 3 years of employment before retirement. B) If an employee has at least 15 years of service with an eligible employer, an additional catch-up contribution may be permitted. C) TSAs are available to all eligible employees of Section 501(c)(3) organizations who adopt such a plan. D) An eligible employee may be able to use both a special catch-up provision and an over-age-50 catch-up provision in the same year.

Answer is A. The special catch-up provision for eligible Section 403(b) participants allows a maximum additional $3,000 per year elective deferral until a lifetime maximum catch-up is reached. The ability to double the worker contribution in the last three years before the retirement plan document's normal retirement age applies to governmental 457 plans.

Which of the following reasons for an early distribution from a qualified retirement plan is NOT an exception to the 10% penalty? A) It is a distribution for higher-education costs. B) The plan owner becomes totally and permanently disabled. C) The distribution is made to a beneficiary of the account due to the owner's death. D) It is made after separation from service from an employer-sponsor of the plan after age 55.

Answer is A. \The exception from the 10% early distribution penalty for distributions for higher-education costs only applies to IRA distributions. LO 6.3.1

Which of the following beneficiaries is entitled to roll over a post-death distribution from a qualified plan into an IRA? A) The surviving spouse of the participant B) All three choices are correct C) The surviving mother of the participant D) The oldest surviving child of the participant

Answer is B. A spouse beneficiary can roll the distribution over into an IRA and treat it as the spouse's own; a nonspouse beneficiary can use a direct trustee-to-trustee transfer of the distribution into a specially titled inherited IRA. LO 6.2.1

In considering whether to convert a traditional IRA to the Roth IRA form, which of the following is a valid consideration? A) It is generally advantageous if the converted assets will remain in the Roth IRA for a relatively short time period before withdrawal. B) If the source of payment for taxes due upon conversion comes from an outside source, it generally is advantageous to convert. C) If the taxpayer files as married filing separately and thereby splits income, it generally makes sense to convert. D) If the taxpayer anticipates being in a lower tax bracket at date of distribution from the Roth IRA, it generally makes sense to convert.

Answer is B. The Roth IRA yields greater after-tax benefits than a traditional deductible IRA if the front-end tax due upon conversion is paid from funds outside the Roth IRA and an equivalent amount of funds is, thereby, available for investment. LO 5.4.2

Which of the following statements is NOT a requirement for the beneficiaries of a trust to be treated as a designated beneficiary of a qualified plan or an IRA? A) The trust is irrevocable at the participant's death. B) The beneficiary of the trust is named on the decedent's retirement account as a named beneficiary. C) The trust is valid under state law. D) The appropriate documentation is provided to the plan administrator.

Answer is B. The beneficiaries of the trust must be named (identified) in the trust instrument. Only the trust needs to be named as a beneficiary on the account. LO 6.4.1

Julie attained age 70½ on November 1, 2020, and will correctly take her first required minimum distribution (RMD) from her qualified plan by April 1, 2021. Her qualified plan balance for which year is used to calculate the RMD she must receive by December 31, 2021? A) The plan balance minus the April 1 distribution she receives in 2021 B) The plan balance at the end of 2020 C) The plan balance at the end of 2021 D) The plan balance at the end of 2019

Answer is B. The postponed 1st-year RMD is based on the participant's plan or IRA account balance as of the end of the year preceding the first distribution year. Her first distribution year is 2020 because she turned 70½ in 2020. The RMD for each year is based on the balance at the end of the previous year. Because the first distribution year is 2020, the balance for the 2020 RMD calculation is the account balance at the end of 2019. The second RMD, which will be for the year 2021, is based on the account balance at the end of 2020. This is the year in question. LO 6.2.2

Which of the following retirement plans, maintained by an eligible employer, would also permit the employer to establish a SIMPLE IRA? A) Money purchase pension plan B) Union plan bargained in good faith C) Section 403(b) plan D) Traditional Section 401(k) plan

Answer is B. To establish a SIMPLE (IRA or Section 401(k)), an employer cannot generally maintain another retirement plan. However, a plan bargained in good faith with a union is allowed because Congress is more worried about powerful businessowners using a retirement plan in a discriminatory manner.

Claudia's simplified employee pension (SEP) plan balance is $60,000. She wants to know her options for taking a loan from her SEP plan to pay some college expenses for her daughter, Caroline. Which of the following statements is CORRECT? A) SEP plan loan repayments must be in level installments payable at least quarterly over a 5-year period. B) Because Claudia is 100% vested in the SEP plan, she may borrow up to $50,000 from the plan. C) Claudia may not make a loan from her SEP plan account. D) Claudia may borrow up to 50% of her SEP account balance to pay for Caroline's college expenses because she is 100% vested in the account contributions.

Answer is C. A SEP plan is a type of IRA. A participant is not permitted to borrow from a SEP plan. LO 6.3.2

All of the following statements regarding the basic provisions of a Section 457 plan are correct except A) distributions from a Section 457 plan are not subject to an early withdrawal penalty. B) the contribution limit is doubled in the 3 years before an individual's retirement. C) a Section 457 plan is a qualified plan of governmental units or agencies, and non-church-controlled, tax-exempt organizations. D) in 2020, an individual who has attained age 50 may make additional catch-up contributions of up to $6,500.

Answer is C. A Section 457 plan is not a qualified plan. This plan is a deferred compensation plan that may be established by governmental units or agencies and non-church-controlled, tax-exempt organizations. Section 457 plans have special catch-up rules. LO 6.3.1

Sherry, who is currently age 50, made only one contribution during her lifetime to her Roth IRA in the amount of $5,000 in 2014. If she were to receive a total distribution of $6,500 from her Roth IRA in 2020 to take a vacation, how would she be taxed? A) Because Sherry waited five years, the distribution will be classified as a qualified distribution and will not be taxable or subject to the 10% early distribution penalty. B) Because Sherry waited five years, the distribution will be classified as a qualified distribution and will not be taxable, but will be subject to the 10% early distribution penalty. C) Although Sherry waited five years, the distribution will not be classified as a qualified distribution, it will be taxable to the extent of earnings, and it will be subject to the 10% early distribution penalty on the taxable amount. D) Although Sherry waited five years, the distribution will not be classified as a qualified distribution and will be fully taxable and will be subject to the 10% early distribution penalty.

Answer is C. A distribution from a Roth IRA is not subject to taxation if it is a qualified distribution or to the extent that it is a return of the owner's contributions or conversions to the Roth IRA. A qualified distribution is one that meets both of the following tests: The distribution was made after a five-year holding period. The distribution was made for one of the following reasons:Owner has attained age 59½Distribution was made to a beneficiary or the estate of the owner on or after the date of the owner's deathDistribution was attributable to the owner's disabilityDistribution was for a first-time homebuyer expense purchase The 10% early withdrawal penalty only applies to a distribution from a Roth IRA that is includable in gross income. The 10% early withdrawal penalty also applies to a nonqualified distribution, even if it is not then includable in gross income, to the extent it is allocable to a conversion contribution made within the five-year period beginning with the first day of the individual's taxable year in which the conversion contribution was made. LO 5.4.2

Geraldine participates in a Section 403(b) plan at work. Three years ago, she borrowed $5,000 from the plan. She has had an outstanding loan balance of $1,000 the past year, and 6 months ago she made the final payment of $1,000 to pay off that loan. Her vested account balance is currently $300,000. What is the maximum allowable loan amount she can take from the plan this year? A) $150,000 B) $145,000 C) $49,000 D) $50,000

Answer is C. Generally, the limit on loans from qualified plans is 50% of the vested account balance, up to a maximum loan of $50,000. The maximum loan must be reduced by any loan balance the participant had in the 1-year period preceding the loan. Geraldine's maximum allowable loan is $49,000. LO 6.3.2

Under the IRA minimum distribution rules, if the IRA account owner dies before distribution payments begin, what occurs? A) The funds revert to the government if no beneficiary is named. B) If the beneficiary is a named individual who is not the spouse, the beneficiary must distribute the entire account over a period not exceeding five years. C) The beneficiary can begin receiving distributions based on the beneficiary's individual life expectancy. D) All of these occur.

Answer is C. If no beneficiary is named, the funds revert to the account owner's estate and are distributed according to the will or the state's intestacy laws. The spouse has the option of rolling over the IRA to the beneficiary-spouse's own account, and a nonspouse beneficiary may use a direct trustee-to-trustee transfer of the IRA into an inherited IRA. A nonspouse named beneficiary is not required to distribute the entire IRA balance within five years. LO 5.3.1

Mark attained age 70½ this year. He does not plan to retire from his position with Big Trucks, Inc., until his birthday on December 1, when he is 72. Mark is a 3% shareholder in Big Trucks. When must Mark begin to receive required minimum distributions (RMDs) from his qualified retirement plan at Big Trucks? A) Because Mark is still employed by Big Trucks, he is not required to take his first RMD until December 31 of the year he actually retires from Big Trucks. B) Mark is not required to receive his first RMD until December 31 of the year following his actual retirement date from Big Trucks. C) Mark is not required to begin his RMDs until April 1 of the year following his actual retirement from Big Trucks. D) Because Mark is a shareholder in Big Trucks, he must receive his first RMD by April 1 of next year.

Answer is C. Participants in qualified plans, Section 403(b) plans, and governmental Section 457 plans may defer the required beginning date until April 1 following the year of retirement, if the participant continues to work after attaining age 70½. If the employee-participant owns more than 5% of the business sponsoring the retirement plan, the RMD may not be deferred but must be taken by April 1 of the year after the employee attains age 70½. LO 6.2.1

The 20% mandatory withholding requirement applies to distributions from all of the following except A) qualified plans. B) Section 457 plans. C) IRAs. D) Section 403(b) plans.

Answer is C. The 20% mandatory withholding requirement does not apply to distributions from traditional IRAs, SIMPLE IRAs, or SEP IRAs. LO 6.3.2

George is a plumbing contractor and has implemented a retirement plan for his employees. The plan must cover all employees who are at least age 21 and have worked for George for three of the last five years (including part-time employees). Contributions must be made for employees who earned at least $600 in the prior year. The plan can exclude union members if they have their own retirement plan. Which type of plan has George selected? A) Section 403(b) plan B) SIMPLE IRA C) Simplified employee pension (SEP) plan D) SARSEP plan

Answer is C. The requirement that contributions must be made for employees who earned at least $600 in the prior years identifies the plan as a SEP plan.

Which of the following statements is NOT correct regarding the conversion of a traditional IRA to a Roth IRA? A) An amount in a traditional IRA may be transferred to a Roth IRA maintained by the same trustee. B) An amount distributed from a traditional IRA can be rolled over to a Roth IRA within 60 days of the distribution. C) The IRA owner's modified adjusted gross income (MAGI) cannot exceed $100,000 in the year of the conversion. D) An amount in a traditional IRA may be transferred in a trustee-to-trustee transfer from the trustee of the traditional IRA to the trustee of the Roth IRA.

Answer is C. There is no MAGI limit for a taxpayer in the year in which there is a conversion. LO 5.4.2

Which of the following retirement plans, maintained by an employer, would also permit an eligible employer to establish a SIMPLE? A) Section 403(b) plan B) SEP plan C) Section 457 plan D) Section 401(k) plan

Answer is C. To establish a SIMPLE, an employer cannot maintain another plan. However, a Section 457 plan is a nonqualified deferred compensation plan; therefore, it does not constitute a plan for purposes of establishing a SIMPLE.

Jerry and Barbara recently filed for divorce after 25 years of marriage. The property settlement approved by the court included an award to Barbara of half of Jerry's vested benefit in his defined benefit pension plan. This was done via the drafting and implementation of a qualified domestic relations order (QDRO). Which of the following is an implication of the QDRO for Jerry and Barbara? A) Barbara's benefit is subject to an additional 10% penalty if received before her age 59½. B) When Jerry retires, Barbara's benefit is taxable to her. C) Barbara's benefit upon receipt is not subject to income tax. D) Under QDRO rules, Barbara is not eligible to roll over the distribution to an IRA.

Answer is C. distribution by a qualified retirement plan to an alternate payee who is a spouse or former spouse of the participant is taxable to the spouse, if it is made pursuant to a QDRO. The distribution to Barbara pursuant to a QDRO is not subject to the 10% penalty for early distribution. Barbara may roll over the distribution to an IRA. LO 6.3.1

All of the following statements regarding simplified employee pension (SEP) plans are correct except A) the major advantage is the simplicity of the plan. B) SEP plans can be established by any form of business entity. C) employer contributions are discretionary. D) all part-time employees can be excluded.

Answer is D. A SEP plan must cover any employee who is at least 21 years old and who worked for the employer, even on a part-time basis, for three of the preceding five years. In addition, contributions must be made on behalf of any eligible employee whose compensation for the year is at least $600 (2020). These provisions make most part-time employees eligible to participate.

All of the following retirement plans permit employees to make elective deferrals except A) Section 401(k) plans. B) SIMPLE 401(k)s. C) profit-sharing plans with Section 401(k) provisions. D) SEP plans.

Answer is D. Employee elective deferrals are permitted in Section 401(k) plans, profit-sharing plans with Section 401(k) provisions, and SIMPLEs, but not in SEP plans.

Jake has named a trust as the beneficiary of his qualified retirement plan. The trust beneficiaries include his four children, ages 4 through 18, and his nephew, age 30. For the trust beneficiaries to be treated as the designated beneficiaries of the plan, the trust must meet all of the following requirements except A) the trust must be valid under state law. B) the trust must be irrevocable or become irrevocable when Jake dies. C) the appropriate documentation has been provided to the plan administrator. D) the trust beneficiaries must not be identifiable from the trust instrument.

Answer is D. For the trust beneficiaries to be treated as designated beneficiaries, the beneficiaries must be identifiable from the trust instrument. LO 6.4.1

Which of the following is NOT correct regarding qualified preretirement survivor annuities (QPSAs)? A) QPSA payments are to begin no later than the month in which the participant would have reached the earliest retirement age under the plan. B) QPSA payments must be the actuarial equivalent of not less than half of the participant's vested account balance as of the date of the participant's death. C) To waive the QPSA benefit for a married participant's spouse, the participant and the spouse must consent on a notarized written form. D) The QPSA payable to the surviving spouse must be equal to the benefit that would have been payable to the participant.

Answer is D. QPSA payments are not required to equal the benefit that would have been payable to the participant at retirement but must be the actuarial equivalent of not less than half of the participant's vested account balance as of the date of the participant's death. LO 6.4.2

Which of the following descriptions of a regular rollover from a qualified plan to a traditional IRA is CORRECT? A) The rollover amount to the IRA is limited to $6,000 (2020). B) It generally must be completed within 90 days of the date of distribution from the previous plan. C) Amounts rolled over are taxable according to rules governing the source of contribution. D) A 20% withholding tax applies in the event of the employee-participant's physical possession of the amount rolled over.

Answer is D. The answer is a 20% withholding tax applies in the event of the employee-participant's physical possession of the amount rolled over. In a regular rollover, the recipient physically receives a check, made payable to the recipient, for the eligible rollover distribution from the plan trustee. Under this method of distribution, the issuer must withhold 20% of the proceeds for federal income tax. LO 6.1.1

Jane Paschal has contributed $1,000 each year to a Roth IRA, beginning with an initial payment of $500 on December 31, 2015. She wants to know when she can begin making qualified distributions. Which one of the following statements represent what you should tell her? A) Qualified higher education expenses are one of the reasons a withdrawal may be a qualified distribution. B) After December 31, 2019, the five years will have elapsed, and she could begin making qualified distributions as soon as she attains age 59½. C) Any distributions for medical expenses in excess of 10% would qualify as a tax-free distribution after satisfying the five-year holding period even if she has not attained age 59½. D) Any distribution she takes after January 1, 2020, will meet the five-year requirement.

Answer is D. The clock started on January 1, 2015, so five years will have elapsed on January 1, 2020. A Roth IRA owner is required to hold the account for a minimum of five years to qualify for tax-free distributions. In addition, the owner must be at least age 59½, dead, disabled, or withdrawing up to $10,000 of Roth IRA earnings for qualified first-time homebuyer expenses. LO 5.4.2

The trustee for Debra Bennett's qualified retirement plan is calculating her required minimum distribution for 2020. The trustee should use the plan balance for A) December 31, 2021. B) December 31, 2020. C) December 31, 2018. D) December 31, 2019.

Answer is D. The trustee should use the plan balance for December 31 of the year preceding the distribution year—in this case, December 31, 2019. LO 6.2.2

David has been calculating his retirement savings needs. He has $250,000 of assets set aside now. His goal is to have $800,000 in 15 years. How much more does he need to save, assuming a 7% annual after-tax rate of return throughout this period? (Round to the nearest dollar.) A) $689,758 B) $110,242 C) $289,957 D) $550,000

B. Calculating the future value of David's current retirement assets: −$250,000 (PV) 15 years from now (n) at a 7% return (i) equals $689,758 (FV). This leaves a difference of $110,242 ($800,000 − $689,758) to be saved by David. LO 8.3.2

Which of the following statements regarding the tax effects of converting a traditional IRA to a Roth IRA is CORRECT? A) The amount converted is not considered a taxable distribution from the IRA to the extent the distribution does not represent a return of basis. B) The 20% premature distribution penalty applies if the owner is less than age 59½. C) The 10% premature distribution penalty applies if the owner is less than age 59½. D) The converted amount is treated as a taxable distribution from the IRA to the extent the distribution does not represent a return of basis.

Answer is D. When a traditional IRA is converted to a Roth IRA, the converted amount is treated as a taxable distribution to the extent the distribution does not represent a return of basis and is included in the owner's gross income. A penalty does not apply for amounts converted from a traditional IRA to a Roth IRA, regardless of the owner's age. However, the amount that was taxable at the conversion will be subject to the 10% early distribution penalty if it is withdrawn within five years of the conversion and the withdrawal does not qualify for an exception to the 10% early withdrawal penalty. This rule protects against people converting to a Roth and then taking a distribution as a way of escaping the 10% early withdrawal penalty.LO 5.4.2

Assuming the account holder is age 40, which of the following withdrawals from a traditional IRA is subject to the 10% early withdrawal penalty? A) $10,000 donated directly to a qualified 501(c)(3) charity B) $10,000 to pay for medical expenses in excess of the 10% (2020) of AGI threshold C) $10,000 to pay for a qualified first-time home purchase D) $10,000 to pay for college tuition for either the account holder or a dependent

Answer is a. For tax purposes, the account holder must be at least age 72 to make a qualified charitable distribution from an IRA. The other answer options are expressly exempt from the 10% early withdrawal penalty, regardless of the account holder's age. LO 5.3.1

Which of the following statements is NOT correct about Roth 401(k) accounts? A) Unlike traditional Roth IRA accounts that have phaseouts based upon income, there are no income restrictions applicable to participation in a Roth 401(k). B) Roth 401(k) accounts allow larger contributions than Roth IRAs. C) Even if a Roth 401(k) participant already has a Roth IRA account, a new five-year clock is required for the Roth 401(k). D) Just as with any Roth IRA account, there are no required minimum distributions (RMDs) that must be made from a Roth 401(k) account.

Answer is d. Roth 401(k) accounts, just as with traditional 401(k) accounts, have RMD rules that apply, meaning that distributions must start in the year the participant reaches age 72 (unless they are still working). This can be avoided by rolling the Roth 401(k) over into a Roth IRA since there are no RMDs with Roth IRAs. There is a new clock that is started for a Roth 401(k) account, even if the participant already has a Roth IRA account. However, if a participant transfers the Roth 401(k) into a Roth IRA, then the Roth IRA clock will be the one that applies, not the Roth 401(k) clock. LO 5.4.3

Joe and Tammy, both age 45 with a combined AGI of $200,000, have become good clients of Frank, a CFP® professional, since coming to Frank several years ago for help in developing a plan to retire at age 55. As part of the plan, an ongoing investment plan was established, and Joe and Tammy have been dedicated to saving the amount called for in the plan to reach their goal. Lately, Frank has noticed Joe's and Tammy's investment preference has become far more conservative than when the plan for early retirement was developed. What should Frank do next as it relates to the retirement plan? A) Frank should discuss the change in investment preference with Joe and Tammy and revise the retirement calculations if Joe and Tammy's risk tolerance has changed. B) Frank should recommend tax-free municipal bonds because of Joe and Tammy's combined AGI. C) Frank should recommend a portfolio of government bonds for added security. D) Frank should discourage Joe and Tammy from becoming more conservative because they are still relatively young.

Assuming a lower rate of return in the retirement plan calculations may impact the amount of ongoing investments necessary to reach Joe's and Tammy's previously stated retirement goal. Without first learning the reason behind the investment preference change, it would be premature to encourage Joe and Tammy to continue to assume more risk or for Frank to recommend government bonds or municipal bonds. LO 8.2.1

Carol has an additional retirement need of $30,000 annually in today's dollars. She will retire in 15 years and projects a retirement period of 20 years. Carol believes she can achieve a 6% after-tax rate of return and is assuming a 4% annual rate of inflation. She has accumulated $175,000 toward her retirement plan. What lump-sum amount should Carol have accumulated over the next 15 years to support her retirement income need? A) $503,707 B) $907,144 C) $873,553 D) $732,144

B. Carol's total retirement fund needed to support her desired standard of living is $907,144, calculated as follows: Carol's first year retirement income need is $54,028. PV = -$30,000 i = 4 n = 15 FV = $54,028 The total capital required to support this need for 20 years is $907,144. In BEGIN mode (the client will make annual withdrawals at the beginning of each year) PMT = $54,028 n = 20 i = 1.9231 [(1.06 ÷ 1.04) - 1] × 100 PVAD = -$907,144 LO 8.3.2

Paul estimates he will need a $75,000 annual income in today's dollars when he retires 10 years from now. He assumes a 3% annual rate of inflation, a 5% after-tax rate of return on his investments, and a 20-year retirement period. Using the level payment approach, how much will Paul need to save in a single annual payment at the end of each year to fund his retirement need? A) $100,794.00 B) $134,331.53 C) $120,880.49 D) $75,000.00

B. Paul will need to make 10 level annual payments of $134,331.53 at the end of each year. Step 1: What will it take to produce $75,000 of purchasing power in 10 years? $75,000 PV 3, I 10, N FV Answer: $100,793.7285 Step 2: How much do they need at retirement to pay a serial payment of $100,793.7285 at the beginning of each year for 20 years? $100,793.7285, PMT 20, N [(1.05 ÷ 1.03) - 1] × 100, I (This number is 1.9417) 0, FV Shift Mar (To get into the Begin Mode) PV Answer is $1,689.607.5595 Step 3: How much needs to be saved as a level payment at the end of each year until retirement so the account will have $1,689,607.5595? $1,689,607.5595, FV 10, N 5, I O, PV Shift, MAR (To change to the End Mode) PMT Answer $134,331.53 LO 8.3.2

To qualify for disability income benefits under Social Security, a worker must have an impairment that A) is expected to result in death within 6 months. B) is expected to last at least 12 months or result in death. C) is related solely to alcoholism. D) is related solely to drug addiction.

B. To qualify for Social Security disability benefits, a person must suffer an impairment that is expected to Medicare tax is assessed on an unlimited amount of compensation. LO 7.1.1

This year, Martin received a lump-sum distribution from his qualified retirement plan. The distribution consisted entirely of his employer's stock, which has a fair market value of $100,000 on the date of distribution to Martin. The adjusted basis of the stock to the trustee of the plan was $70,000. Assuming Martin does not sell the stock this year, what amount is included in Martin's gross income as a result of the distribution? A) $30,000 B) $0 C) $100,000 D) $70,000

Because the distribution is a lump-sum distribution of employer stock, the net unrealized appreciation (NUA) rules apply. Under the NUA rules, the adjusted basis of the stock to the trust ($70,000) is included in Martin's gross income in the year of the distribution and is treated as ordinary income. LO 6.1.2

Which of the following describe differences between a tax-advantaged retirement plan and a qualified plan? IRA-funded employer-sponsored tax-advantaged plans may not incorporate loan provisions. Employer stock distributions from a tax-advantaged plan do not benefit from NUA tax treatment.

Both statements are correct. IRA-funded employer-sponsored tax-advantaged plans are SEPs, SAR-SEPs, and SIMPLE IRAs. LO 6.1.2

Which of the following plans would NOT be subject to annual additions limit? A) Age-weighted profit-sharing plan B) Traditional profit-sharing plan C) Defined benefit pension plan D) Target benefit pension plan

C. A defined benefit pension plan is not subject to the annual additions limit. Each of the other plans listed is a defined contribution plan which is subject to the annual additions limit. LO 8.1.1

What is the maximum amount of Social Security benefits that may be subject to income taxation? A) 50% B) 35% C) 85% D) 0%

C. If someone has provisional income in excess of $32,000 for unmarried taxpayers and $44,000 for married taxpayers, up to 85% of Social Security benefits may be subject to income taxation. These numbers are not indexed. Also, the threshold is $0 for people who file their return as married filing separately (MFS) if they are not living apart. LO 7.2.2

Which of the following retirement plans, maintained by an eligible employer, would also permit the employer to establish a SIMPLE IRA? A) Section 403(b) plan B) Money purchase pension plan C) Section 457 plan D) Traditional Section 401(k) plan

C. To establish a SIMPLE (IRA or Section 401(k)), an employer cannot generally maintain another retirement plan. However, a Section 457 plan is technically a nonqualified plan and, therefore, does not constitute a plan for purposes of establishing a SIMPLE of any type. LO 8.1.1

Which of the following statements with respect to simplified employee pension (SEP) contributions made by an employer is CORRECT? A) Contributions are capped at $19,500 for 2020. B) Contributions are subject to FICA and FUTA. C) Contributions are currently excludible from the employee's gross income. D) Contributions are subject to income tax withholding.

Contributions are currently excludible from the employee's gross income. Employer contributions to a SEP are not subject to FICA, FUTA, or income tax withholding. The SEP contribution limit for 2020 is the lesser of 25% of covered compensation or $57,000.

Jill and Mark are celebrating their 20th wedding anniversary, receiving 20-year watches from their employers, and reaching full retirement age (FRA), all on the same day. Which of the following statements correctly describe the Social Security benefit Jill is eligible to receive? A) 100% of her own benefit B) 75% of Mark's benefit C) 85% of her own benefit D) 100% of Mark's benefit

Currently, the spouse of a Social Security recipient is entitled to 50% of the recipient's primary insurance amount (PIA), subject to a family maximum, as long as the spouse is of full retirement age (FRA) or between age 62 and FRA for reduced benefits. The spouse will take the greater of either 100% of their own benefit or 50% of their spouse's benefit. LO 7.2.1

If a businessowner-client is of an older age, near his retirement date, and just establishing a qualified plan, which of the following plans would generally NOT be advantageous to the owner? A) An age-weighted profit-sharing plan B) New comparability plan C) Traditional defined benefit pension plan D) Traditional profit-sharing plan

D. A traditional profit-sharing plan would not allow the plan contributions to be skewed for the benefit of the older owner. All of the other plan choices listed would allow greater contributions for the older owner. LO 8.1.2

Which of the following retirement plans would be appropriate for a general partnership with stable cash flows? A) ESOP B) Stock bonus plan C) Section 403(b) plan (TSA) D) Age-weighted profit-sharing plan

D. An ESOP and stock bonus plan may only be established by an S or C corporation. A Section 403(b) plan (TSA) is only available to certain tax-exempt organizations. An age-weighted profit-sharing plan is the only plan appropriate for a general partnership from among the choices given. LO 8.1.2

A covered individual or his survivor is entitled to which of the following benefits under Social Security if the worker is only in a currently insured status? A) Surviving spouse not caring for dependent child benefit B) The worker's own retirement benefits C) Spousal retirement benefits D) Surviving child's (dependent) benefit

D. An individual who is only currently insured under Social Security (6 credits of coverage in preceding 13-quarter period) is entitled to a surviving child's benefit. This is the case provided the child is under age 18, over age 18 and disabled by a disability that began before age 22, or under age 19 and a full-time elementary or secondary school student who is not married and was dependent on the deceased parent. LO 7.1.1

Richard participates in a traditional defined benefit pension plan at work. His projected monthly benefit under the plan is $1,000. If the plan provides life insurance for Richard, the death benefit payable under the policy is limited to A) $215,000. B) $17,500. C) $52,000. D) $100,000.

Defined benefit plans use the 100 times test for determining whether they comply with the incidental benefit rules. Under this test, the death benefit cannot exceed 100 times the participant's projected monthly benefit (in this instance, $100,000). LO 8.2.2

Delaying receipt of benefits (for example until age 70) will result in all of the following except A) receipt of delayed retirement credits. B) permanently reduced benefit amount. C) larger survivor benefits for the surviving spouse. D) higher annual cost-of-living adjustments.

Delaying receipt of benefits will allow you to earn delayed retirement credits, and your future cost-of-living adjustments will be higher because they will be based on a higher base. Also, the increased base will ultimately be used to determine survivor benefits. LO 7.2.1

Which of the following statements best describes the definition of disability in qualifying for disability benefits under Social Security? A) An individual must not be able to engage in any substantial, gainful activity, and the impairment must be expected to last at least 12 months or result in death. B) An individual must not be able to engage in the work of any occupation for which she is trained. C) An individual must be able to engage in work as specified by a Social Security Administration examiner. D) An individual must not be able to engage in the work associated with his last employment position.

Disability, for the purpose of Social Security, means an individual is so severely physically or mentally disabled that the person cannot engage in any substantial, gainful activity, which must be expected to last at least 12 months or result in death. LO 7.1.1

Elaine is currently age 76 and scheduled to take another distribution from her former company's qualified retirement plan later this year. Her account balance in the plan as of December 31 last year was $320,000. Under the Uniform Lifetime Table, the divisor is 22. However, Elaine's actual life expectancy is only 16 years. What is the amount, if any, of Elaine's required minimum distribution (RMD) from this plan for this year? A) $14,545 B) $53,333 C) $0, because Elaine is over age 70½ D) $20,000

Elaine's required minimum distribution this year is $14,545, calculated as follows: $320,000 ÷ 22.0 = $14,545. She must calculate her required distribution using the Uniform Lifetime Table and not her actual life expectancy (nor by using the difference between projected and actual life expectancy as the applicable divisor). This is actually advantageous for the owner because the original owner's RMD is usually based on the owner's age and the age of someone 10 years younger than the owner. This is true even if the account owner is not married. The only time the assumed life expectancy in a RMD calculation for the original account owner is different than this is when the original owner is actually married to someone who is more than 10 years younger. In that case, the RMD is based on the actual ages of the married couple. LO 6.2.2

In 2020, James earned $6,000 from employment subject to Social Security taxation between January 1 and September 30. He was then unemployed for the remainder of the year. How many credits of coverage did James earn for the year 2020? A) 2 B) 1 C) 4 D) 3

For 2020, a worker receives 1 credit for each $1,410 in annual earnings on which Social Security taxes are withheld up to a maximum of 4 credits annually. All 4 credits may be earned in the same calendar quarter. LO 7.1.1

Guy and Dotty, who are both age 42, are married and file a joint tax return. Their modified adjusted gross income (MAGI) for 2020 is $130,000. Dotty has already made a $6,000 contribution to her traditional IRA and has also made a $2,000 contribution to their son's Coverdell Education Savings Account this year. What is the maximum amount that may be contributed, if any, to a Roth IRA for Guy and Dotty this year given these facts? A) $2,000 B) $0 C) $11,000 D) $6,000

For 2020, the maximum combined contribution to traditional and Roth IRAs (for an owner younger than age 50) is $6,000 per person annually. Dotty has already contributed the maximum amount for her traditional IRA, but they can make a further contribution of $6,000 to a Roth IRA for Guy. The $2,000 Coverdell contribution is never relevant to any retirement account contribution. Their AGI is below the Roth IRA phaseout range for a married couple ($196,000-$206,000) in 2020. LO 5.4.1

Maryellen, age 63, is receiving Social Security retirement benefits. She also works part time, and her earnings are $10,000 more than the earnings limit. Her Social Security retirement benefits this year will be reduced by A) $7,500. B) $5,000. C) $10,000. D) $0.

Maryellen has not reached full retirement age, so she is subject to the retirement earnings test. Her Social Security benefits will be reduced by $1 for every $2 in earnings over the applicable limit. LO 7.2.2

John and Mary, both age 49, are married and file a joint income tax return for the current year (2020). John is self-employed as an engineering consultant and reports $120,000 of Schedule C net income and pays $16,955 in self-employment tax. Mary is not employed outside the home. What is the maximum deductible IRA contribution John and Mary can make this year? A) $12,000 B) $7,000 C) $6,000 D) $0

Neither John nor Mary is an active participant in an employer-sponsored retirement plan, qualified retirement plan, SEP plan, SIMPLE, or Section 403(b) plan; therefore, they can contribute and deduct $12,000 ($6,000 each) to traditional IRAs for the current year (2020). LO 5.1.2

Scott and Gayle, who are both age 45, are married and file a joint income tax return for the current year. Scott is a self-employed architect who earns $110,000 of Schedule C income and pays $15,543 in self-employment tax. Gayle is not employed outside the home. What is the maximum deductible IRA contribution Scott and Gayle can make, if any, for 2020? A) $6,000 B) $14,000 C) $12,000 D) $0

Neither Scott nor Gayle is an active participant in an employer-sponsored retirement plan. Therefore, they can establish a traditional IRA for Scott and a spousal IRA for Gayle and contribute a deductible total of $12,000 ($6,000 each) to traditional IRAs for 2020.

Diane and Amanda, who are 28-year-old twins, are partners in a computer software consulting firm with 20 employees. The average age of their employees is 25 and their length of employment averages three years. The firm is profitable and enjoys stable cash flows. Diane and Amanda have talked about a retirement plan as an employee benefit and do not want to assume the investment risk of the plan. The partners do wish to encourage employees to make elective deferrals. Of the following retirement plan options, which is best suited for their business? A) A profit-sharing plan with a Section 401(k) provision B) An eligible Section 457 plan C) A Section 403(b) plan D) A stock bonus plan

Of the plans listed, the profit-sharing plan with a Section 401(k) provision is the best choice for Diane and Amanda. This plan allows for both employer and employee contributions. A Section 403(b) plan may only be adopted by a Section 501(c)(3) organization. An eligible Section 457 plan may only be adopted by a private tax-exempt organization or a state or local governmental organization. A partnership cannot use a stock bonus plan because this form of business does not issue stock. LO 8.2.1

All of the following workers are covered by the Social Security system except A) self-employed workers. B) members of the armed services. C) railroad workers covered under the federal Railroad Retirement Act. D) employees of tax-exempt organizations.

Railroad workers covered under the federal Railroad Retirement Act are not covered by Social Security but are covered by Medicare if they meet those rules separately. LO 7.1.1

Distributions from IRAs must begin by April 1 of the year following the year in which an individual reaches age A) 72. B) 75. C) 59½. D) 65.

Penalties are imposed upon distributions that commence before age 59½ or after April 1 following the year the account owner turns age 72. LO 5.3.1

Max, age 47, has been participating in his employer's SIMPLE IRA for one year. If he withdraws $1,000 from this plan this year and the withdrawal is not covered by an exception to the penalty tax on premature withdrawals, he will owe a penalty tax of A) $100. B) $250. C) $400. D) $0.

Premature withdrawals made from a SIMPLE IRA within two years of initial participation are subject to a penalty tax of 25%. Max's penalty would be $250.

Brent and Carol have an adjusted gross income (AGI) of $40,000, and they receive a combined Social Security benefit of $15,000. They have no tax-exempt income. What percentage of their Social Security benefit will be subject to taxation? A) 100% B) 50% C) 85% D) 0%

Provisional income = $47,500. This is over the $44,000 threshold, so 85% of their Social Security benefit will be subject to taxation. LO 7.2.1

George was born in 1962. His full retirement age (FRA) for Social Security purposes is A) 65. B) 62. C) 67. D) 70.

The FRA for individuals born in 1960 and later is 67. LO 7.2.2

Higher-income earners will have a Social Security retirement income replacement ratio that is A) There is no relationship between the Social Security income replacement ratios of high and low earners. B) the same as low-income earners. C) higher than low-income earners. D) lower than low-income earners.

Replacement rates, or the amount of one's paycheck that is replaced by Social Security retirement benefits, favor lower-income earners by replacing about 90% of their (very low) earnings. Higher earners will see only a 26% replacement. LO 7.2.1

Ronald, age 44, works for two private tax-exempt employers. One has a Section 403(b) plan and one maintains a nongovernmental Section 457 plan. If Ronald defers $10,000 into the Section 403(b) plan in 2020, what is the maximum amount he may defer into the Section 457 plan? A) $47,000 B) $19,500 C) $13,500 D) $9,000

Ronald can separately defer the maximum of $19,500 (2020) into the Section 457 plan because Section 457 plan limits are not aggregated with the Section 403(b) plan limits.

Claude's ex-wife, Sara, has secured a qualified domestic relations order against his Section 401(k) plan. What are Sara's rollover options if she takes a lump-sum distribution of her share of Claude's retirement plan account? A) Sara may rollover the distribution to her own qualified plan, Section 403(b) plan, Section 457 plan, or IRA. B) Sara may rollover the distribution to her own qualified plan, Section 457 plan, or an IRA, but not to a Section 403 (b) plan. C) Sara may rollover the distribution to her own qualified plan, Section 403(b) plan, or Section 457 plan, but not to an IRA. D) Sara may rollover the distribution to her own qualified plan, Section 403(b) plan, or an IRA, but not to a Section 457 plan.

Sara may rollover the distribution to her own qualified plan, Section 403(b) plan, Section 457 plan, or IRA. LO 6.5.1

Social Security provides individuals with protection against which of the following risks? A) Inflation risk B) Longevity risk C) Market risk D) All of these

Social Security, for many retirees, is their only source of income that protects against inflation risk, market risk, and longevity risk.

Which one of the following types of employers can offer either a 403(b) plan or a 457(b) plan? A) Churches and qualified church-controlled organizations may offer either or both a Section 403(b) and a Section 457(b) plan. B) Any employer with less than 1,000 employees can offer either or both a Section 403(b) and a Section 457(b) plan. C) A state university or college could offer either or both a Section 403(b) and a Section 457(b) plan. D) Any employer that can offer a Section 403(b) plan could also sponsor a Section 457(b) plan.

State and local governments and organizations exempt from federal income tax are eligible to offer Section 457 deferred compensation plans, but churches and church-controlled organizations cannot. No for-profit organizations can offer either of these plans.

Social Security began as a program to provide retirement income but has been expanded to provide all of the following income except A) survivor benefits to spouses at age 60‒61. B) survivor benefits to children under age 19. C) survivor benefits to spouse caring for a child under 19. D) disability.

Survivor benefits are provided to a spouse caring for a child under 16 or disabled. Age 16 is not "sweet 16" for the surviving spouse because when the youngest child turns 16, the survivor benefits ceased for the widow(er) caring for the deceased's child. LO 7.2.1

George and Mabel each put $6,000 into their respective IRAs. George's employer does not provide a qualified retirement plan. Mabel participates in a 401(k) plan at work. Their AGI is $199,000 in 2020, and they file jointly. How much of their IRA contributions will be deductible? A) $4,200 B) $0 C) $6,000 D) $12,000

The IRA rules allow an IRA deduction for individuals who are not active participants but whose spouses are, in some cases. However, that option is phased out if the couple's AGI is between $196,000 and $206,000 in 2020. With a combined AGI of $199,000, George would be able to deduct $206,000 − $199,000 = $7,000; ($7,000 ÷ $10,000) × $6,000 = $4,200. LO 5.1.2

What amount is used to determine a participant's actual Social Security retirement benefit? A) Primary insurance amount (PIA) B) Permitted disparity limit C) Covered compensation limit D) QJSA

The PIA is used to determine a participant's actual Social Security retirement benefit. The PIA is derived from the worker's average indexed monthly earnings (AIME), which is based on lifetime earnings history. LO 7.1.1

Which of the following is correct regarding qualified joint and survivor annuities (QJSA) provided for in qualified pension plans? A) QJSA provisions may never be waived. B) The QJSA amount payable to the surviving spouse may not be less than 50% of the amount of the annuity payable during the life of the participant. C) The maximum QJSA payable to a surviving spouse may not exceed 150% of the amount of the annuity payable during the life of the participant. D) QJSA provisions are never applicable to a Section 401(k) plan.

The QJSA amount payable to the surviving spouse may not be less than 50% (nor greater than 100%) of the amount of the annuity payable during the life of the participant. The QJSA may be waived with written notarized consent of the participant's spouse. A Section 401(k) plan may be subject to QJSA requirements unless certain requirements are met. LO 6.4.2

All of the following are considerations for converting distributions from qualified plans or a traditional IRA to a Roth IRA except A) the Roth IRA conversion becomes more appropriate the longer the period of distributions. B) one advantage of a conversion to the Roth IRA is that the Roth IRA will not be subject to required minimum distributions (RMD) during the life of the original owner. C) any portion of an IRA can be converted to a Roth IRA. D) the Roth IRA conversion is more appropriate when the income tax rate is lower at the time of distribution than at the time of conversion.

The Roth IRA conversion is more appropriate when the income tax rate is the same or higher at the time of distribution than at the time of conversion. LO 5.4.3

For tax-exempt employers that do not want to implement a Section 457 plan and desire a plan funded strictly by employee elective deferrals, a good alternative would be A) a Section 403(b) plan. B) a profit-sharing plan. C) a SEP plan. D) a SARSEP plan

The Section 403(b) plan, like the Section 457 plan, can be used as an employee-deferred contribution plan. Certain tax-exempt employers can implement Section 403(b) plans. With a SEP plan or a profit-sharing plan, there are also employer contribution considerations. New SARSEP plans can no longer be established.

Caitlin has determined she has a future value of retirement savings need of $1,157,140. If she retires in 16 years and achieves an 8% after-tax annual return on her investments, what amount of level end of year annual deposit is required to fund this need? A) $38,159 B) $20,285 C) $35,332 D) $23,878

The annual level end of year deposit required is $38,159, calculated as follows: In END mode FV = $1,157,140 n = 16 i = 8 PMT = −38,158.8576 LO 8.3.2

Which of the following statements describes a component of calculating the lump-sum capital amount necessary to fund the projected income need over the retirement need period? Adjust or inflate the projected first-year retirement income need (expressed in present value dollars) to future dollars at the time of retirement. Calculate the total retirement fund needed (lump-sum capital amount) to meet the projected income demands. To do this, the planner must calculate the present value of an annuity due using an inflation-adjusted rate of return.

The answer is both I and II. LO 8.3.1

All of the following are primary assumptions in any retirement needs calculation except A) the projected rate of annual investment return. B) the anticipated annual rate of inflation. C) the number of dependents a client will have at retirement. D) the client's age at retirement and anticipated life expectancy.

The answer is the number of dependents a client will have at retirement. There are three primary assumptions that are made in any retirement needs analysis calculation. They are the anticipated annual rate of inflation; the projected rate of annual investment return; and the client's age at retirement and anticipated life expectancy. LO 8.3.1

All of the following statements regarding the capital preservation approach to retirement needs analysis calculations are correct except A) this approach is the most commonly used approach in making retirement needs analysis calculations. B) a person using this approach will need to accumulate a larger retirement fund than if he had used the capital utilization approach. C) this approach is most appropriate for affluent clients. D) this approach assumes the client will spend the income from his retirement fund and leave the principal intact.

The capital preservation approach assumes the client will spend the income from his retirement fund but leave the principal intact. This approach requires the client to accumulate a larger retirement fund than the capital utilization approach does, and it is generally appropriate for more affluent clients. Most retirement needs analysis calculations use the capital utilization approach. LO 8.3.1

David is age 47 and qualifies for a Section 457 plan through his job with the state. David's salary is $37,500. What is the maximum salary deferral that David can contribute to the Section 457 plan in 2020? A) $9,375 B) $19,500 C) $6,500 D) $13,500

The dollar limit applicable to Section 403(b), 457, and 401(k) plans is $19,500 in 2020.

For which of the following plans does the employee bear the investment risk in the plan? SIMPLE 401(k) Traditional profit-sharing plan SEP IRA SIMPLE IRA

The employee bears the investment risk for all of these plans.

When does the maximum family benefit limitation NOT apply to reduce total Social Security benefits payable? A) When both the husband and the wife receive a retirement benefit based on their own employment record B) When the wife receives a spousal benefit based on the husband's employment record C) When a divorced spouse qualifies for benefits because she is caring for a dependent child of the worker-participant D) When the husband receives a spousal benefit based on the wife's employment record

The maximum family benefit limitation does not apply when both the husband and wife receive a retirement benefit based on their own (or each) employment record. It does apply to reduce total Social Security benefits payable in the other situations. LO 7.2.2

Gordon is the fiduciary for a traditional IRA. He has several different investments available to him to invest the IRA assets. All of the following investments are permitted investments for a traditional IRA except A) corporate bonds. B) a real estate investment trust (REIT). C) stock in Bottle, Inc., which is an S corporation. D) mutual funds.

Traditional IRAs cannot own S corporation stock, as an IRA is not a permissible shareholder under the S corporation rules. All of the other choices are permitted investments.

Gordon is the fiduciary for a traditional IRA. He has several different investments available to him to invest the IRA assets. All of the following investments are permitted investments for a traditional IRA except A) stock in Bottle, Inc., which is an S corporation. B) a real estate investment trust (REIT). C) mutual funds. D) corporate bonds.

Traditional IRAs cannot own S corporation stock, as an IRA is not a permissible shareholder under the S corporation rules. All of the other choices are permitted investments. LO 8.2.1

Blake, age 72, is required to take substantial required minimum distributions (RMDs) from his qualified retirement plan. He has no current need for the cash and has established traditional IRAs with his children as beneficiaries and wishes to deposit the distributions in equal amounts to each IRA within 60 days of the distributions to eventually benefit his children. Which of the following statements regarding Blake's rollover of the RMDs is CORRECT? A) Required minimum distributions may not be rolled over, but Blake may make equivalent contributions within 60 days of his RMD to the traditional IRAs. B) A good plan is for Blake to roll over the distribution within 60 days after receipt. C) Required minimum distributions may not be rolled over. D) Because Blake is over age 70½, he may not roll over the RMDs to a traditional IRA, but he may roll over the RMDs to a Roth IRA.

While Blake may not roll over the distributions, if he has earned income, he may make contributions of the lesser of $7,000 ($6,000 regular contribution and $1,000 catch-up in 2020) annually, or 100% of his earned income to a Roth IRA (or split the contribution over multiple Roth IRAs).

Which of the following is CORRECT about a Roth IRA? A) As with conventional IRAs, distributions from a Roth IRA must begin by April 1 of the year following the year the participant reaches age 72. B) If a nonqualifying distribution is made before age 59½, the principal is subject to the 10% penalty, but it is not considered to be taxable income. C) An individual can contribute $6,000 annually to a regular IRA and $6,000 annually to a Roth IRA. D) Withdrawals of earnings up to $10,000 from a Roth IRA for the purchase of a first home can be penalty free if the five-year holding period has been met.

Withdrawals of earnings from a Roth IRA are not penalized under these circumstances if the five-year holding period has been met. LO 5.4.1

Fully insured status generally requires a measure of employment covered by Social Security (OASDI) that is 40 A) years. B) credits of coverage. C) calendar quarters. D) months.

answee is B. To qualify for most benefits under Social Security, an individual must be in fully insured status. This is defined as one credit per year since age 21 with a minimum of 6 and a maximum of 40 credits of covered employment, where an individual has earned above a specified dollar amount. LO 7.1.1

Which of the following statements regarding prohibited transactions by a fiduciary or an individual associated with traditional IRA accounts is CORRECT? Generally, if an individual or the individual's beneficiary engages in a prohibited transaction with the individual's IRA account at any time during the year, it will not be treated as an IRA as of the first day of the year. If an individual borrows money against an IRA annuity contract, the individual must include in gross income the fair market value of the annuity contract as of the first day of the tax year. Selling property to an IRA by a fiduciary or an individual owner of the IRA is not prohibited. A 50% penalty will be assessed against an IRA owner who borrows money against her IRA.

answer 1 and 2 Statements I and II correctly describe prohibited transactions. Statement III is incorrect. Selling property to an IRA by a fiduciary or an individual owner of the IRA is a prohibited transaction. Statement IV is incorrect. The individual may have to pay the 10% additional tax on premature distributions.

Jeff and Julie, both age 50, are married and are both fully insured under Social Security. They have two children. Amy is 24 and was injured in an auto accident when she was 21 leaving her disabled and dependent on her parents for her care. Brad is 17 and a senior in high school. Jeff's mother, Lisa, is age 66 and lives with the couple but is not a dependent for tax purposes. If Julie dies, who may receive a survivor benefit under Social Security based upon her fully insured status? Amy Brad Jeff Lisa

answer 1, 2, and 3 Amy, Jeff, and Brad will receive benefits. Amy became disabled before age 22 and will receive a benefit, as will Jeff, who is now her caregiver. Brad will receive a benefit because he is an unmarried child under age 19 and still in high school. Because Lisa is not a dependent, she is not eligible for a benefit. LO 7.2.2

Which of the following statements is(are) CORRECT regarding the Social Security Windfall Elimination Provision (WEP)? The WEP is designed to reduce Social Security retirement benefits for any worker who is eligible for retirement benefits from multiple employers. An example of a worker who may be subject to the WEP is one who works for a government agency or a foreign employer. The WEP applies to all Social Security benefits for which a family member or dependent of a covered worker may qualify. In the case of a worker subject to the WEP, the provision is applied by reducing the formula used to calculate a worker's primary insurance amount (PIA), resulting in a lower benefit.

answer 2 and 4 The WEP does not apply simply if a worker is eligible for retirement benefits from multiple employers. It may apply to a worker who qualifies for a Social Security retirement or disability benefit but who also earns a pension from an employer who does not withhold Social Security taxes. An example of such an employer could be a state or local government agency that had opted out of the Social Security retirement system; a public school employee is some states; or an employer in a foreign country. Under this provision, the formula used to calculate a worker's PIA is reduced, resulting in a lower benefit. Social Security benefits are designed to pay a higher percentage of career average earnings for lower-paid workers than are paid to higher earning workers. Statement III is incorrect because the WEP does not apply to spousal or survivors benefits under Social Security. The Government Pension Offset would apply to Social Security spousal or survivor benefits when the non-Social Security worker is the spouse or the survivor of a fully covered retired or deceased worker. LO 7.2.2

Which of the following statements is(are) CORRECT regarding rollovers from qualified plans or IRAs? Distributions from qualified plans and IRAs require 20% mandatory withholding for federal income taxes if a trustee-to-trustee direct transfer is not used to execute a rollover. A taxpayer is limited to one rollover in a one-year period (on a 365-day basis) unless the rollover is a trustee-to-trustee direct transfer. A distribution from a qualified plan may not be rolled over to a governmental Section 457 plan. If a qualified plan participant has an outstanding loan from a qualified plan upon separation from service, the participant may roll over the loan into a rollover IRA as long as loan repayments continue at least quarterly.

answer 2 only Statement I is incorrect because IRA distributions do not require 20% mandatory federal income tax withholding. Statement III is incorrect because a rollover is permitted from a qualified plan to a governmental Section 457 plan. Statement IV is incorrect because loans are not permitted from an IRA and may not be rolled over from an IRA to a qualified plan. LO 6.1.1

Ryan wants to take a distribution from his SIMPLE 401(k) account balance from his previous employer and deposit it in an IRA at his local banking institution. Which of the following statements regarding his transfer is(are) CORRECT? The distribution from the SIMPLE 401(k) plan is not subject to mandatory 20% withholding. A direct transfer from Ryan's SIMPLE 401(k) to an IRA is not subject to the mandatory 20% income tax withholding requirement.

answer 2 only The answer is II only. Statement I is incorrect. SIMPLE 401(k) plans are qualified plans and are subject to mandatory 20% income tax withholding for a distribution that is not a direct trustee-to-trustee transfer. When there is a direct transfer of a distribution from a qualified plan to an IRA, the mandatory 20% withholding rule does not apply.

Under the Social Security system, immediate survivor income benefits based on a deceased worker's primary insurance amount and coverage are available to which of the following persons? A surviving spouse, age 55, caring for the worker's 13-year-old child Unmarried, dependent children under age 18 Unmarried children who become disabled before age 22 A surviving divorced spouse, age 62, who has not remarried and was married to the decedent for more than 10 years

answer all of them Each of these persons is eligible for survivor's benefits.

What is the earliest age at which a currently insured worker may claim Social Security retirement benefits based on her own work history? A) Age 62 B) Age 67 C) Retirement benefits are not available for a currently insured worker D) Any age over age 59½, but benefits will be reduced

c. The worker must be fully insured to qualify for retirement benefits based on her own work history. Reduced spousal retirement benefits are available as early as age 62. Reduced survivor benefits are available as early as age 60 unless the surviving spouse is disabled, then reduced survivor benefits are available as early as age 50. LO 7.2.2

Margaret is a 29-year-old attorney with her own law practice. She has hired four part-time employees over the past five years to assist her. Each of these employees works approximately 200 hours per year, earning an average annual salary of $4,000. Margaret would like to establish a retirement plan that would allow her to begin saving for her own retirement, with little administrative costs. Which one of the following plans would be most appropriate for Margaret? A) SEP plan B) Section 457 plan C) SIMPLE IRA D) Traditional Section 401(k) plan

A SIMPLE IRA would be the most appropriate plan because it involves little administrative costs and would meet Margaret's retirement plan goals. Notice the workers are making more than $600 and less than $5,000. If a SEP would be chosen, she would have to contribute for employees who made more than $600; are at least age 21; and who have worked for her for three of the preceding five years.

Which of the following is a type of traditional defined benefit pension plan? A) A target benefit pension plan B) A money purchase pension plan C) A fully insured Section 412 (e)(3) pension plan D) An employee stock ownership plan (ESOP)

A fully insured Section 412(e)(3) pension plan is a type of traditional defined benefit pension plan. The other types of plans noted are defined contribution plans, including the money purchase pension plan. LO LO 2.2.2

The Acme Corporation has six owners, ranging in age from 30 to 60 years old, and 25 rank-and-file employees. The owners want to adopt a qualified retirement plan that will allow them to maximize the contributions to the owners' accounts and to minimize the contributions to the accounts of the rank-and- file employees. Which of the following plans would best meet the owners' needs? A) Keogh plan B) Section 401(k) plan C) New comparability plan D) Age-based profit-sharing plan

A new comparability plan would allow the owners to divide the participants into two classes based on their compensation levels, and to allocate different contribution levels to the classes. An age-based profit-sharing plan wouldn't meet their objectives because the owners' ages are significantly different. Section 401(k) plans are subject to discrimination testing, and a Keogh plan is inappropriate because the owners are not self-employed.

Netcyber, Inc., is a five-year-old C corporation that has experienced dramatic growth during three of the past five years. There are currently 68 employees. The two owners, who are 28 and 31 years old, wish to establish a retirement plan. Which plan would probably be the best for them? A) Target benefit pension plan B) Age-weighted profit-sharing plan C) Profit-sharing plan D) Money purchase pension plan

A profit-sharing plan would be most appropriate, as the owners could choose when to make contributions. Also, because the owners are young, they would have more time to receive contributions on their own behalf. A money purchase pension plan or target benefit pension plan would not be appropriate because this young company would not want to commit a large sum of money every year. The company probably would want the ability to reinvest the money in the firm if needed. The target benefit pension and age-weighted profit-sharing plans are also designed for older owner-employees.

The major difference between a stock bonus plan and a traditional profit-sharing plan is that A) a stock bonus plan generally distributes benefits in employer stock, not cash. B) the maximum annual additions limit is higher for a stock bonus plan. C) in a stock bonus plan, the employer can only contribute employer securities to the plan. D) a profit-sharing plan is subject to nondiscrimination requirements while a stock bonus plan is not.

A stock bonus plan generally distributes benefits in employer stock, not cash. The maximum annual additions limit is $57,000 in 2020 for all defined contribution plans. In a stock bonus plan, the employer can contribute either cash or employer securities to the plan. The contributions are determined in a variety of ways (a percentage of either profits or covered payroll). Stock bonus plans are subject to the same nondiscrimination requirements as profit-sharing plans. LO 3.1.1

The actual deferral percentage (ADP) test is generally applicable to which of the following? Traditional Section 401(k) plans Money purchase pension plans SIMPLE 401(k)s

Answer: 1 only The ADP test applies only to traditional Section 401(k) plans. The test does not apply to SIMPLE 401(k)s or money purchase pension plans.

Which of the following is CORRECT regarding IRS Form 5500? A) All of these statements are correct. B) Filing an IRS 5500 is an ERISA requirement. C) The IRS 5500 is known as the employer's annual return/report to the IRS of an employee benefit plan. D) A simplified version, Form 5500-EZ, is available for certain small employers.

All of these statements are correct regarding IRS Form 5500.

A Section 401(k) plan allows plan participants the opportunity to defer taxation on a portion of compensation simply by electing to defer compensation to the plan instead of receiving it in cash. Which of the following rules apply to Section 401(k) elective deferrals? Section 401(k) elective deferrals are immediately 100% vested and cannot be forfeited. A Section 401(k) plan may allow in-service distributions. Plans may allow loan provisions. The maximum elective deferral contribution for 2020 is $19,500, with an additional $6,500 catch-up for individuals age 50 or older.

All of these statements are correct.

ERISA requires reporting and disclosure of defined benefit plan information to the PBGC. plan participants. the IRS. the DOL.

All of these statements are correct.

Which of the following are reasons a small business might choose the SIMPLE over a Section 401(k) plan? A) All of these are reasons. B) The employer expects that it could not satisfy the Section 401(k) nondiscrimination test. C) Because a SIMPLE is less costly to operate, it is generally the better choice if the employer is not concerned about the design constraints of the plan. D) A Section 401(k) plan would be top heavy (benefits for key employees will exceed 60% of total benefits), and the employer wants to minimize employer contributions.

All of these statements are reasons a small business might choose the SIMPLE over a Section 401(k) plan. LO 4.1.1

Which plan is appropriate for a business with a fluctuating cash flow and an owner who is significantly older than the rank-and-file employees? A) Age-weighted profit sharing B) Defined benefit C) Cash balance D) Target benefit

An age-weighted formula can be used by a profit-sharing plan. This arrangement will work best for a business with a fluctuating cash flow and key employees who are older than the other employees. Pension plans (i.e., cash balance plans and target plans) are subject to minimum funding requirements and may create a problem for a business with a fluctuating cash flow, even though they will maximize contributions for older employees. A service-based formula will favor employees who have many years of service with the business. LO LO 3.2.1

An employer's annual contributions to a qualified defined contribution plan must be within statutory limits regarding the allowable amount. An employer's annual contribution to a defined contribution plan is limited to ____% of the firm's covered payroll. A) 10 B) 25 C) 15 D) 20

An employer's annual contributions to a qualified defined contribution plan must be within statutory limits regarding the allowable amount. An employer's annual contribution to a defined contribution plan is limited to 25% of the firm's covered payroll.

An increase in which of the following factors will increase plan costs for a defined benefit pension plan? A) Employee turnover B) Mortality C) Life expectancies D) Investment returns

An increase in employee life expectancies will increase the potential costs of the plan, so life expectancies have a direct impact on costs. LO 2.1.1

How are target benefit pension plans similar to money purchase pension plans? The participant's final account balance determines the actual retirement benefit. The maximum annual additions limitations are the same. The employer bears the investment risk. An actuary determines the employer contribution each year.

Answer 1 and 2 A target benefit pension plan is a type of defined contribution plan and is similar to a money purchase pension plan in that the participant's account balance determines the actual retirement benefit, the maximum annual additions limitation is the same, and the employee bears the investment risk. An actuary is used only at inception of the target benefit pension plan, not annually. 2.3.1

Tax implications of cash balance pension plans include which of these? Employer contributions on behalf of employees grow tax deferred. Employer contributions to the plan are deductible when made. Annual retirement benefits are limited to $285,000 (2020). There are no penalties for distributions before age 59½.

Answer 1 and 2 Statements I and II are correct. Section 415(b) limits the benefits provided under the plan to the lesser of $230,000 per year (2020) or 100% of the participant's highest consecutive three-year average compensation. Distributions from the plan must follow the rules for qualified plan distributions and may be subject to penalty for early distribution. LO 2.2.1

Which of the following statements regarding top-heavy plans is CORRECT? An accelerated vesting schedule is used when a defined benefit pension plan is top heavy. A qualified plan is considered top heavy if it provides more than 50% of its aggregate accrued benefits or account balances to key employees. Top-heavy defined benefit plans must provide a minimum benefit of 2% per year of service for up to 10 years (20%) for all non-key employees. For a top-heavy plan, a key employee is an employee who owns more than 3% of the employer with compensation greater than $125,000 (2020).

Answer 1 and 3 An accelerated vesting schedule is used when a defined pension benefit plan is top heavy. A defined contribution plan always requires an accelerated vesting schedule. A qualified plan is considered top heavy if it provides more than 60% of its aggregate accrued benefits or account balances to key employees. A key employee is an employee who, at any time during the plan year, is the following: greater than a 5% owner; a greater than 1% owner with compensation greater than $150,000 (not indexed); or an officer of the employer with compensation greater than $185,000 in 2020.

Which of the following statements are advantages for the employer-sponsor of a cash balance pension plan? A certain level of plan benefit is guaranteed by the PBGC. The employer bears the investment risk in the plan. Cash balance pension plans are less expensive to administer for the employer than traditional defined benefit pension plans. Retirement benefits may be inadequate for older plan entrants.

Answer 1 and 3 Statements I and III are correct. A cash balance pension plan has several advantages and disadvantages. LO 2.2.1

In a Section 401(k) plan, which of the following must be considered in complying with the maximum annual additions limit? Employee after-tax contributions Catch-up contributions for an employee age 50 or older Dividends paid on employer stock held in a Section 401(k) plan Employer qualified non-elective contributions

Answer 1 and 4 Statement I is correct. Employee after-tax contributions are counted against the annual additions limit. Statement II is incorrect. Catch-up contributions for an employee age 50 or older are not counted against the annual additions limit. Note that catch-up contributions are also not taken into account for purposes of ADP testing or plan limits. Statement III is incorrect. Earnings on plan investments are not taken into account when computing the maximum annual additions limit. Statement IV is correct.

Jeff wants to establish a qualified plan for his business to provide employees of the company with the ability to save for retirement. Which of the following plans is a qualified plan? Profit-sharing plan Simplified employee pension (SEP) plan SIMPLE IRA Top-Hat plan

Answer 1 only Of the plans listed, only the profit-sharing plan is a qualified plan. The SIMPLE IRA and the SEP plan are tax-advantaged plans, and the Section 457 plan is a nonqualified plan.

Qualified retirement plans should do which of the following? They must meet specific vesting requirements. They have special tax advantages over nonqualified plans. They must provide definitely determinable benefits. They require an annual profit to allow funding for the plan.

Answer 1, 2, 3 Qualified plans must meet specific vesting schedules. Qualified plans are preferred to nonqualified plans because of the special tax advantages enjoyed by qualified plans. Qualified retirement plans must offer definitely determinable benefits. An annual profit is not required for a qualified plan to be funded.

Which of the following is NOT required to be distributed to qualified plan participants on an annual basis? Summary Plan Description (SPD) Form 5500 Summary Annual Report (SAR) Summary of Material Modification (SMM)

Answer 1, 2, 4 Only a SAR, summarizing the basic information included in the Form 5500 series, must be provided to plan participants each year within nine months of the end of the plan year. An SPD must be provided automatically to all plan participants within 120 days after the plan is established or 90 days after a new participant enters an existing plan, not annually. Form 5500 must be filed with the IRS and DOL annually, but is not required to be provided annually to plan participants; however, participants have a right to see the full Form 5500 if needed and requested. An SMM, explaining any substantive changes that occurred to the SPD within the past year, must be issued as needed.

Mincher Publications just implemented a safe harbor Section 401(k) plan. Which of the following may be avoided with a safe harbor Section 401(k) plan? ADP test ACP test Top-heavy rules Coverage tests

Answer 1, 2, and 3 Safe harbor Section 401(k) plans are not required to comply with ADP, ACP, or top-heavy rules. However, all plans must meet the general coverage nondiscrimination rules.

A savings incentive match plan for employees (SIMPLE) can be which of these? It can be established as an IRA. It can be established as a Section 401(k) plan. It can be established as a Section 403(b) plan. It can be offered by employers who have 100 or fewer employees.

Answer 1, 2, and 4 Employers with more than 100 employees (who earn more than $5,000) may not offer a SIMPLE. In addition, the employer may not usually offer any other type of qualified retirement plan in conjunction with a SIMPLE. Union plans and governmental 457 plans are the exceptions to the employer having a second active plan. However, this is not to be applied to a test question unless specifically mentioned in the question. Basically, can a SIMPLE have a second active plan for its workers? No, because that is as far as the test is going. What about a union plan or a 457 for qualified state and local government employers? Well, okay, for those rare exceptions, but the question would have to bring up one of these rare exceptions. The point is that SIMPLEs were intended to be the only active plan for the small employer. Congress was afraid the employer would try to be discriminatory with the other retirement plan, so the rules was made that there could not be another retirement plan. Then governmental 457 plans and union plans snuck in, but these rare exceptions are not a part of the question unless specifically mentioned.

Which of the following is CORRECT about the permitted disparity (Social Security integration) rules for defined benefit plans? A plan that provides a benefit for wages up to the integration level, plus a higher benefit for wages that exceed the integration level, is an integrated defined benefit excess plan. A plan that provides that an employee's benefit otherwise computed under the plan formula is reduced by a fixed amount or formula amount in relationship with the person's Social Security benefit is an integrated defined benefit offset plan. Covered compensation is the average of the participant's compensation not in excess of the taxable wage base for the three-consecutive-year period ending with or within the plan year. The base benefit percentage is determined by calculating the benefits provided by the plan based on compensation below the integration level, and expressing these benefits as a percentage of compensation below the integration level.

Answer 1, 2, and 4 Statements I, II, and IV are correct statements about the definition of integrated defined benefit excess plans, the definition of integrated defined benefit offset plans, and how to determine the base benefit percentage. Option III is incorrect because covered compensation means the average Social Security taxable wage base over the last 35 years.

Which of the following statements regarding a top-heavy plan is CORRECT? A top-heavy defined benefit pension plan must provide accelerated vesting. A top-heavy plan is one that provides more than 50% of its aggregate accrued benefits or account balances to key employees. A top-heavy defined pension benefit plan must provide a minimum benefit accrual of 2% multiplied by the number of years of service (up to 20%). For a top-heavy defined contribution plan, the employer must make a minimal contribution of 3% of annual covered compensation for each eligible non-key employee. If the contribution percentage for key employees is less than 3%, the contribution percentage to non-key employees can be equal to the key employees' percentage.

Answer 1, 3, 4 Only Statement II is incorrect. A top-heavy plan is one that provides more than 60% of its aggregate accrued benefits or account balances to key employees. Notice that 60% exactly does not make a plan top heavy.

Which of the following statements regarding fully insured Section 412(e)(3) plans is CORRECT? A fully insured Section 412(e)(3) plan is a type of defined benefit plan. All fully insured Section 412(e)(3) plans must meet minimum funding standards each plan year. This type of plan is not required to be certified by an enrolled or licensed actuary. A fully insured plan is inappropriate for an employer who cannot commit to regular premium payments.

Answer 1, 3, 4 Statements I, III, and IV are correct. Section 412(e)(3) plans are a type of defined benefit plan. This type of plan is not required to be certified by an enrolled or licensed actuary. A fully insured plan is inappropriate for an employer who cannot commit to regular premium payments. Statement II is incorrect. Fully insured Section 412(e)(3) plans must only meet minimum funding standards if there is a loan outstanding against the insurance policy funding the plan. LO LO 2.2.2

The plan trustee of an employee stock ownership plan (ESOP) can borrow funds to purchase employer stock. cannot borrow funds to purchase employer stock. is using leverage when the trustee borrows money to provide contribution to the plan. uses loan proceeds to purchase stock of the employer from the corporation itself.

Answer 1, 3, and 4 An ESOP may or may not have provisions for borrowing. An ESOP that has provisions for borrowing is known as a LESOP (leveraged ESOP). The employer receives the loan proceeds immediately. The loan is paid back from employer contributions to the ESOP.

Qualified retirement plans have which of the following characteristics? They are subject to ERISA requirements. They offer tax-deferred earnings to employees. They can discriminate in favor of highly compensated employees. They provide tax deduction for the employer only when the employee is income taxed on the money.

Answer 1, and 2 Statements I and II are correct. Qualified retirement plans are subject to ERISA requirements and provide tax deferral on investment earnings for employees. While qualified plans in general can provide different levels of benefits to different classes of employees, qualified plans cannot "discriminate in favor of highly compensated employees" in the sense that there is a legal limit to the amount of the difference. As long as the difference is inside the legal limits, the plan is not discriminatory (by definition). Qualified retirement plans provide an immediate tax deduction on employer contributions.

Which of the following are agencies that administer and ensure compliance with the federal laws that apply to qualified retirement plans? Department of Labor (DOL) Employee Retirement Income Security Agency (ERISA) Internal Revenue Service (IRS) Pension Benefit Guaranty Corporation (PBGC)

Answer 1,3,4 Under the reporting and disclosure requirements of the Employee Retirement Income Security Act of 1974 (ERISA), annual reports and summary plan descriptions are filed with the IRS (which provides copies to the DOL). An annual premium payment form is filed with the PBGC. If the defined benefit plan is not in compliance with funding requirements, the PBGC can terminate the plan. If the defined benefit plan is not in compliance with funding requirements, the PBGC can terminate the plan. LO 1.1.1

Mary, age 56, earns a salary of $125,000 and works for a company that sponsors a Section 401(k) plan. The plan allows her to contribute up to 15% of her salary each year, up to the annual Section 401(k) plan elective deferral limit. The company matches her contribution dollar-for-dollar, up to 3% of compensation. Because Mary would like to retire within the next five years, she is concerned about having a sufficient retirement benefit from the Section 401(k) plan. Based on life expectancy tables, Mary is expected to live until age 85. Which of the following factors can affect Mary's retirement benefits from her Section 401(k) plan? The number of years of service multiplied by a benefit formula The participant's investment selections The value of the participant's account balance at retirement

Answer 2 and 3 Factors affecting retirement benefits from a Section 401(k) plan are the participant's investments selections and the value of the participant's account balance at retirement. Statement I is related to defined benefit plans, not defined contribution plans.

Which of the following statements are disadvantages for the employer-sponsor of a cash balance pension plan? A certain level of plan benefit is guaranteed by the PBGC. The employer bears the investment risk in the plan. Cash balance pension plans are less expensive for the employer than a traditional defined benefit pension plan. Retirement benefits may be inadequate for older plan entrants.

Answer 2 and 4 Statements II and IV are correct. A cash balance pension plan has several advantages and disadvantages. Advantages include the following: * A certain level of plan benefits is guaranteed by the PBGC. *There are significant cost savings for the employer as compared to the traditional defined benefit pension plan. Disadvantages include the following: *The employer bears the risk of poor investment performance. *The retirement benefits may be inadequate for older plan entrants. *If the plan is a converted traditional defined benefit pension plan, the lump-sum payout at the employee's retirement date may be considerably less under the cash balance formula. This is a disadvantage to workers and an advantage to employers. LO 2.2.1

Under which of the following circumstances is a target benefit plan the most appropriate choice for small-business owners? To simplify and reduce the cost of eliminating a defined benefit plan (without termination) by amending it into a target plan Where the employer wants to provide larger retirement benefits for key employees who are significantly older than the other employees To meet the employer's goal of maximizing deductible contributions to provide benefits for older, highly compensated employees Where the employer is opposed to assuming the investment risk and prefers the simplicity of a separate account plan

Answer 2 and 4. Statements II and IV are correct for the following reasons. Target benefit plans have benefit formulas similar to those of defined benefit plans, which favor employees who are significantly older and higher paid than the average employee of the employee group. Because target benefit plans use separate accounts, the participants bear the risk of the plan's investment performance. Statement I is incorrect because amending a defined benefit plan into a target plan will result in termination of the defined benefit plan. Statement III is not true since a DC plan is limited to 25%, and a DB plan is not limited by a set percentage. Therefore, the DB plan will usually provide a greater tax deduction if an age-weighted plan works best. The key word to Statement III is "maximizing." LO LO 2.3.2

Which of the following legal requirements apply to money purchase pension plans? The normal retirement benefit must be specified in the plan. The plan must provide a definite and nondiscretionary employer contribution formula. Forfeitures can be reallocated to the remaining participants' accounts in a nondiscriminatory manner or be used to reduce employer contributions. A separate employer contribution account must be maintained for each participant.

Answer 2, 3, 4 Statements II, III, and IV correctly state the employer contribution formula rule, the forfeiture reallocation rule, and the separate account rule for money purchase pension plans. The first option states a requirement that applies to defined benefit pension plans but not to money purchase plans.

Tax implications regarding Section 401(k) plans include which of the following? Employee elective deferrals are currently taxable income to the employee. Employee elective deferrals are subject to FICA and FUTA taxes. Employer contributions are deductible by the employer up to 25% of total covered compensation. Employees do not pay federal income taxes on elective deferrals contributed to a Section 401(k) plan until distributed.

Answer 2, 3, and 4 Employee deferrals are not currently taxable income to the employee. All other statements are correct.

Which of the following qualified retirement plans allow unrestricted investment in employer securities? Money purchase pension plans Stock bonus plans Employee stock ownership plan (ESOPs) Traditional profit-sharing plans

Answer 2, 3, and 4 Money purchase pension plans (and other pension plans) allow only 10% contributions in employer securities. All of the other plans are types of profit-sharing plans and may invest 100% in securities.

Which of the following plans is NOT a cross-tested plan? New comparability plan Employer stock ownership plan Age-based profit-sharing plan Stock bonus plan

Answer 2, and 4 Of the plans listed, the employer stock ownership plan and the stock bonus plans are not cross-tested plans. Only the new comparability plan and the age-based profit-sharing plan are cross-tested plans. LO LO 3.2.1

An employer must comply with restrictions on the use of more than one qualified retirement plan where employees participate in both plans. Which of the following are CORRECT about some of these limitations? If a target benefit plan is a fully insured plan and covered under the PBGC, then the target benefit plan is not taken into account in applying the overall limit on deductions. When an employer has more than one plan, the overall contribution limit will be the lesser of 25% of total participants' compensations or the amount required to meet the minimum funding standard of the defined benefit plan. This limit applies only to the extent that the employer contribution to the defined contribution plan or plans is greater than 6%. If an employer sponsors both a defined benefit plan and a defined contribution plan (and the defined benefit plan is exempt from the PBGC), the overall employer limit is the amount necessary to meet the minimum funding requirement of the defined benefit plan. A contribution of up to 6% of compensation is also allowed into the defined contribution plan. Under current law, if a defined benefit plan is covered under the PBGC, then the defined benefit plan is not taken into account in applying the overall limit on employer deductions.

Answer 3 and 4 One exception to the overall deduction limit is for defined benefit plans that are covered by the PBGC. Under current law, if a defined benefit plan is covered under the PBGC, then the defined benefit plan is not taken into account in applying the overall limit on employer deductions. LO LO 2.1.2

Safe harbor Section 401(k) plans are appropriate for employers who want to encourage plan participation among highly compensated employees. want a plan that must be tested annually for coverage requirements. want to provide for a high level of employee elective deferrals without annual discrimination testing. have highly compensated employees whose elective deferrals would be limited in a traditional Section 401(k) plan because of ADP testing.

Answer 3 and 4 Statements III and IV are correct. Safe harbor Section 401(k) plans are appropriate for employers who want a plan that does not need to be tested annually for coverage requirements and encourages plan participation among rank-and-file employees. These plans also ease the employer's administrative burden and costs by eliminating the tests ordinarily applied under a traditional Section 401(k) plan.

Which of the following regarding a SIMPLE is CORRECT? A SIMPLE requires ADP testing of employee elective deferral contributions. SIMPLE IRAs are subject to top-heavy rules. A 25% early withdrawal penalty may apply to distributions taken within the first two years of participation in the SIMPLE IRA plan. The maximum annual elective deferral contribution to a SIMPLE 401(k) is $19,500 (2020) for an employee who has not attained age 50.

Answer 3 only Neither a SIMPLE IRA nor a SIMPLE 401(k) requires ADP testing. A SIMPLE is not subject to top-heavy rules. Statement III is correct. The early withdrawal penalty is 25% for distributions taken within the first two years of participation in a SIMPLE IRA. The maximum elective deferral to a SIMPLE is $13,500 (2020). Employees who have attained age 50 by the end of the tax year will also be eligible for a catch-up contribution of $3,000.

Which of the following is a permitted vesting schedule for a SIMPLE 401(k)? Three-year cliff Two-to-six-year graded 100% immediate vesting Three-to-seven-year graded

Answer 3 only Vesting schedules are not permitted in SIMPLE 401(k)s. Employees are always 100% vested in employer contributions.

In which of the following ways are target benefit pension plans similar to money purchase pension plans? Pension Benefit Guaranty Corporation (PBGC) insurance is required. The actual dollar retirement benefit is guaranteed. The employee bears the investment risk. Each employee has an individual account.

Answer 3, 4 Statements III and IV are correct. PBGC insurance is not available for target benefit or money purchase pension plans. Also, the actual dollar retirement benefit is not guaranteed in either of these plans. LO LO 2.3.2

Which of the following types of qualified retirement plans do NOT allow integration with Social Security? Traditional defined benefit pension plan Money purchase pension plan Profit-sharing plan Employee stock ownership plan (ESOP)

Answer 4 An ESOP may not be integrated with Social Security.

Your client, the chief financial officer of a new company, wishes to install a retirement plan in the company in which the pension benefits to employees are guaranteed by the Pension Benefit Guaranty Corporation (PBGC). Identify the plan(s) that must meet this requirement. (CFP® Certification Examination, released 12/96) Profit-sharing plan Money purchase plan Target benefit plan Defined benefit plan

Answer 4 Only two defined benefit pension plans (defined benefit and cash balance) have PBGC insurance. Note: In the online version of Kaplan Schweser's QBank, the letters preceding the answer choices that appear in the original CFP Board question have been eliminated. The answer choices may not be in the same order as in the original CFP Board-released questions.LO 2.1.1

Joe, age 46, has owned his company for 18 years and wishes to retire at age 70. All of Joe's employees are older than he is and have an average length of service with the company of eight years. Joe would like to adopt a qualified retirement plan that would favor him and reward employees who have rendered long service. Joe has selected a traditional defined benefit pension plan with a unit benefit formula. Which of the following statements regarding Joe's traditional defined benefit pension plan is CORRECT? Increased profitability would increase both Joe's and his employees' pension contributions. A unit benefit formula allows for higher levels of integration than other defined benefit plans. A unit benefit formula rewards older employees hired in their 50s or 60s. A traditional defined benefit pension plan will maximize Joe's benefits and reward long-term employees based on length of service.

Answer 4 Statement I is incorrect. Contributions to defined benefit pension plans are not dependent on the profitability of a company. Statement II is incorrect, because a unit benefit formula plan will not allow higher integration levels. Statement III is incorrect because a unit benefit formula favors workers with longevity. Employees hired in their 50s or 60s will not have significant years of service in the formula at retirement.

ERISA requires reporting and disclosure of plan information to all of the following except A) the Securities and Exchange Commission (SEC) B) the Internal Revenue Service (IRS). C) the Department of Labor (DOL). D) plan participants.

Answer A. ERISA requires reporting and disclosure of plan information by plan sponsors to the IRS, DOL, Pension Benefit Guaranty Corporation (PBGC), and plan participants. LO 1.1.1

Claudia's vested Section 401(k) plan balance is $60,000. She wants to know her options for taking a loan from her plan to pay some college expenses for her daughter, Caroline. Which of the following statements is CORRECT? Claudia may borrow up to $30,000 from her Section 401(k) plan to pay for Caroline's college expenses. A plan loan is generally limited to half of the vested account balance of the plan participant, not to exceed $50,000. All loan repayments for any loan must be in level installments payable at least quarterly. If the rules for a plan loan are not followed, a plan loan may be deemed a taxable plan distribution and may also be subject to the 10% early withdrawal penalty.

Answer All of them All of these statements are correct.

Gary is employed by the city of Great Rapids, and his sister, Julie, works for Big Toys, Inc. Gary participates in a 457 plan and Julie participates in a Section 403(b) plan. Which of the following statements is CORRECT in indicating how their respective plans compare to each other? Both plans are based on contracts with the employer. Both plans allow for the deferral of salary into the plans. Neither plan is a qualified plan. Both plans are subject to limits on the amount that can be contributed to the plan.

Answer All of them Both the Section 403(b) plan (TSA) and the Section 457 plan are based on contracts with the employer (the salary deferral agreement). Both plans allow workers to contribute to the plan, and there are annual limits for the contributions to each plan. Neither plan is a qualified plan.

Which of the following statements regarding Social Security integration and defined contribution plans is CORRECT? The integration level can be less than the taxable Social Security wage base. The maximum permitted disparity will depend on whether the integration level is equal to the taxable wage base or below it. Integration can be used to enhance an owner's contribution to the plan if the owner's compensation is in excess of the Social Security wage base. The integration level cannot be greater than the Social Security taxable wage base.

Answer All of them If the integration level is less than the taxable wage base, the permitted disparity amount may be reduced. Integration can enhance an owner's and/or key employee's contribution rate if the compensation is in excess of the Social Security wage base. A defined contribution plan cannot have integration levels greater than the taxable wage base.

Which of the following is a hybrid retirement plan that uses an actuarially designed benefit formula like that of a traditional defined benefit plan and individual accounts like that of a defined contribution plan? A) Age-weighted profit-sharing plan B) Target benefit pension plan C) Cash balance pension plan D) Money purchase pension plan

Answer B The plan described is a target benefit pension plan. The contribution is derived from an actuarially designed benefit formula in a target benefit pension plan, but once determined, the plan resembles a money purchase pension plan in all other ways. Cash balance pension plans are defined benefit pension plans that promise a benefit based on a hypothetical account balance versus a traditional defined benefit plan, which promises a monthly retirement benefit for life. Age-weighted profit-sharing plans are defined contribution plans that allocate contributions to participants in such a way that when contributions are converted to equivalent benefit accruals (stated as a percentage of compensation), each participant receives the same rate of benefit accrual. A money purchase pension plan does not use an actuarially designed benefit formula. LO LO 2.3.2\

Which of the following is a test that must be met before a hardship withdrawal from a profit-sharing plan? The financial needs test requires the hardship be due to an immediate significant financial need of the participant-employee. The resources test requires that the participant must not have other financial resources sufficient to satisfy the need.

Answer Both Both statements describe the tests that must be met to qualify for a hardship withdrawal. LO 3.1.1

Paul's employer maintains a qualified defined benefit pension plan. There are 100 eligible employees working for the company. What is the minimum number of employees the retirement plan must cover to satisfy the 50/40 test? A) 90 B) 70 C) 40 D) 50

Answer is 40 Under the 50/40 test, a defined benefit plan must cover the lesser of 50 employees or 40% of all eligible employees. In this case, the lesser of 50 employees or 40% of all eligible employees (100) is 40 employees. One way to remember the 50/40 test is the phrase people before percentages (50 people or 40%). Also, note that there are no qualifiers to the types of people. It is not 50 non-highly compensated people. It is just 50 humans who work for the employer.

Tina is a participant in her company's employee stock ownership plan (ESOP). The company transfers 1,000 shares of employer stock at $5 per share to her account. Several years later, when the stock is $11 per share, Tina retires. If she elects to receive the stock in a lump sum at retirement and later sells the stock for $12,000, what are the tax consequences to Tina? A) Tina will have a capital gain of $7,000. B) Tina will be taxed on $11,000 of ordinary income when she receives the stock at retirement. C) Tina will have a capital gain of $1,000. D) Tina will have ordinary income of $12,000 when she sells the stock.

Answer is A Because of the net unrealized appreciation rule, at distribution, Tina will be taxed only on $5,000, the original cost of the stock when it was contributed to the plan. Tina was taxed on the $5,000 basis in the stock when she received her distribution. Therefore, her capital gain will be $7,000 if she sells the stock for $12,000.

This year, its pension actuary informed Gear-It-Up Bicycles, Inc., that its required annual contribution to the company defined benefit pension plan needs to be significantly higher than last year. Which of the following factors could be responsible for this increase in annual contributions? A) Lower-than-expected earnings on plan investments B) Higher-than-expected forfeitures C) Higher-than-expected employee turnover D) Lower-than-expected number of employees nearing normal retirement age

Answer is A. Earnings on investment plans, which were lower than anticipated, will result in an increase to the required annual employer contributions to a defined benefit pension plan. The other factors noted may result in a decrease in plan contributions. LO 2.1.1

A money purchase pension plan is a defined contribution plan in which A) the employer typically contributes a fixed percentage of participant compensation each year. B) the employer must contribute 25% of participant compensation each year. C) the final retirement benefit amount is guaranteed. D) the employer receives no tax deduction for the contribution.

Answer is A. In a money purchase pension plan, the employer typically contributes a fixed percentage of each participant's compensation. The employer does receive a tax deduction for the amount of contribution to the plan. No guaranteed retirement benefit is provided in a defined contribution plan. The contribution is defined, not the benefit. Only defined benefit pension plans provide guaranteed retirement benefits. LO 2.3.1

Great Benx Corporation provides both a defined benefit plan and a money purchase plan for its employees. The defined benefit plan is covered by the PBGC. All employees participate in each plan. If the Section 415 limits apply, how do they apply? A) The Section 415 limits are applied separately for each plan. The annual additions limit for the money purchase plan in 2020 is 100% of the participant's compensation or $57,000, whichever is less. The participant's benefit in the defined benefit plan is limited in 2020 to 100% of the participant's compensation or $230,000, whichever is less. B) Under the Section 415 limits, the total contributions to both accounts of a participant are limited to 25% of the participant's compensation. C) Each participant could receive a maximum contribution of 25% of the participant's compensation for the money purchase plan and a maximum contribution of 100% of the participant's compensation for the defined benefit plan. D) The Section 415 limits no longer apply. These limits were repealed by the Economic Growth and Tax Relief Reconciliation Act of 2001 (EGTRRA) for qualified plan application.

Answer is A. The annual additions limit for the money purchase plan in 2020 is 100% of the participant's compensation or $57,000, whichever is less. The participant's benefit in the defined benefit plan is limited in 2020 to 100% of the participant's compensation or $230,000, whichever is less. The Section 415 limits are applied separately for each plan. LO LO 2.1.2

Which of the following statements regarding target benefit pension plans is NOT correct? A) Each plan participant has an individual account in the plan. B) The ultimate account balance for the employee's account is guaranteed by the plan. C) Target benefit pension plans are a type of defined contribution plan. D) Target benefit pension plans favor older participants.

Answer is B A target benefit pension plan is a type of defined contribution plan and, as in all other defined contribution plans, there is no guaranteed ultimate account balance. The plan favors older participants by making a larger contribution for an older plan entrant than for a younger plan entrant with exactly the same salary. LO LO 2.3.2

Which of the following statements describes a basic provision or use of a savings incentive match plan for a SIMPLE IRA? A) Only employers that average fewer than 200 employees can establish a SIMPLE IRA. B) One contribution formula an employer can use in a SIMPLE IRA is to make a 2% nonelective contribution on behalf of eligible employees. C) A SIMPLE IRA is primarily suitable for large, corporate-type employers. D) A SIMPLE IRA must satisfy special nondiscrimination tests in addition to general rules.

Answer is B. A 2% nonelective contribution formula on behalf of eligible employees is one of two formulas an employer may use in making contributions to a SIMPLE IRA. A 3% match is the other. For a SIMPLE IRA only, the 3% match can be lowered to 1% for two out of five years.

which of the following ways is a target benefit pension plan similar to a traditional defined benefit pension plan? A) Employer contribution is flexible B) Benefits older employees C) Investment returns have no bearing on benefit D) Employee assumes the investment risk

Answer is B. A target benefit pension plan is similar to a traditional defined benefit plan in that they are both designed to benefit older employees. The employer bears the risk in the defined benefit plan only, and the employer contribution is not flexible in either plan. In a target benefit pension plan, the investment returns do affect the final benefit. LO LO 2.3.2

Fernando, age 45, participates in his employer's defined benefit pension plan. This plan provides for a retirement benefit of 2% of earnings for each of his years of service with the company and, given Fernando's projected service of 20 years, will provide him with a benefit of 40% of final average pay at age 65. What type of benefit formula is this plan using? A) A flat benefit formula B) A unit benefit formula C) A discretionary formula D) A flat percentage of earnings formula

Answer is B. Because this formula considers both compensation and years of service, the defined benefit plan is using a unit benefit formula. This type of formula is frequently used to reward continued service with the same employer. LO LO 2.1.2

Which of the following qualification requirements applies to an employee stock ownership plan (ESOP)? A) No more than 50% of plan assets may be invested in employer securities. B) Assets may be invested primarily in qualifying employer securities. C) At retirement, a lump-sum distribution of employer securities is subject to ordinary income tax on the fair market value of the securities. D) Stock is sold via a public offering and the cash received on the sale is contributed to the qualified plan by the employer.

Answer is B. ESOP assets may be invested primarily in employer securities. This is an exception to the normal rule prohibiting more than 10% of qualified plan assets to be held in employer stock or securities. In an ESOP, stock is not sold to the public, but, rather, the plan trustee purchases stock from the employer. A lump-sum distribution of employer securities may be eligible for NUA treatment and LTCG taxation on the NUA portion, rather than ordinary income taxes on the entire distribution.

Gary was just hired by an employer that maintains a SIMPLE IRA for its employees. Which of the following statements regarding Gary's participation in the SIMPLE IRA is CORRECT? A) Gary may only defer $6,000 into the SIMPLE IRA if he is younger than age 50. B) When Gary participates in the plan, he will be 100% vested in his employer's contributions. C) Covered compensation is limited to $285,000 for a SIMPLE IRA in 2020 if the employer elects a 3% match. D) The annual employer match may be limited to 1% of employee compensation each year.

Answer is B. Gary will be 100% vested in his employer contributions to his account. He may defer up to $13,500 (2020) into the SIMPLE IRA if he is younger than age 50. An employer may only limit the matching contribution to 1% of employee compensation in no more than two out of every five years. Effective covered compensation for employees under age 50 is limited to $450,000 ($13,500 ÷ 0.03) for a SIMPLE IRA (2020) in which the employer elects to make 3% matching contributions. The covered compensation limit of $285,000 (2020) does not apply in this instance because there is a special rule for SIMPLE IRAs.

Tom, age 54, is the sole proprietor of a small business. He is interested in adopting a retirement plan for the business. His primary goals are to make large contributions to his own retirement account and to minimize the expense and paperwork associated with the plan. Which of the following retirement plans would you recommend for Tom's business? (He makes $50,000 of self-employment income and has three part-time employees who earn $15,000 each.) A) SEP plan B) SIMPLE IRA C) Section 401(k) plan funded by employee elective deferrals D) Traditional defined benefit pension plan

Answer is B. Qualified plans require a significant amount of expense and paperwork. Because Tom's business is small, a SEP or SIMPLE IRA is preferred. Tom can, however, defer more in a SIMPLE than in a SEP. Based on 2020 plan contribution limits, in the SIMPLE, he can defer $16,500 ($13,500 salary deferral, plus $3,000 as a catch-up contribution) plus receive a 3% match. In a SEP, he would be limited to $9,293 as follows: $50,000Schedule C income-3,533deductible half of self-employment tax (SECA)$46,467net Schedule C SE income× 20%(the 20% is calculated by 0.25 ÷ 1.25 = 20%)$9,293

Three common formulas for determining annual retirement benefits from a defined benefit plan include all of the following except A) the flat percentage of earnings. B) the dollar percentage matrix. C) the flat dollar amount. D) the percentage of earnings per year of service (unit credit).

Answer is B. The dollar percentage matrix formula does not exist. LO LO 2.1.2

What is the maximum annual contribution by an employer to a plan participant's money purchase pension plan account in 2020? A) $19,500 B) Lesser of 100% of employee compensation, or $57,000 C) The amount required to pay the benefit promised without limit D) 25% of the employee's compensation

Answer is B. The maximum annual contribution to the participant's account under the plan is the lesser of 100% of the eligible employee's compensation, or $57,000 (for 2020). The deduction for employer contributions to a defined contribution plan is limited to 25% of aggregate covered compensation. LO 2.3.1

Which one of the following is a correct statement regarding a 401(k) that allows Roth 401(k) plan elections? A) As with a Roth IRA, a Roth 401(k) is not subject to the required beginning date for distributions. B) An employee can contribute $19,500 each to a 401(k) plan and to a Roth 401(k) plan. C) An employee has to aggregate and the maximum employee deferral between Roth 401(k) deferrals and traditional 401(k) deferrals in both plans is $19,500 in 2020.

Answer is C. A Roth 401(k) plan is subject to the minimum distribution rules, including the required beginning date, although the money can be rolled into a Roth IRA to avoid that. Also, one has to aggregate and the maximum employee deferral between a 401(k) plan and a Roth 401(k) plan is $19,500.

Ross, age 75, works for Financial Strategies, Inc. The company has a long-established retirement plan. The plan does not require an actuary or Pension Benefit Guaranty Corporation (PBGC) insurance, but the employer is required to make annual mandatory contributions to each employee's account. What type of retirement plan was established by Financial Strategies? A) Target benefit pension plan B) Traditional defined benefit pension plan C) Money purchase pension plan D) Cash balance pension plan

Answer is C. A money purchase pension plan requires annual mandatory employer contributions to each employee's account, does not require an actuary, and does not require PBGC insurance. The other choices are incorrect: A cash balance pension plan requires an actuary and PBGC insurance. A target benefit pension plan requires an actuary at the inception of the plan, but not on an annual basis. A traditional defined benefit pension plan requires the services of an actuary annually as well as PBGC insurance. LO 2.3.1

Which of the following would NOT be a permitted disparity for a defined benefit plan that uses Social Security integration? A) An excess benefit percentage of 10% if the base percentage is 5% B) An excess benefit percentage of 20%, if the base percentage is 15% C) An excess benefit percentage of 60%, if the base percentage is 30% D) An excess benefit percentage of 40%, if the base percentage is 20%

Answer is C. Base percentage + permitted disparity = excess benefit percentage. The permitted disparity is the base percentage, up to a maximum of 26.25% (0.75% per year for up to 35 years).

Which one of the following requirements is a possible disadvantage of a simplified employee pension (SEP) for an employer? A) Employer contributions to a SEP are subject to payroll taxes. B) A SEP must have a fixed contribution formula that is nondiscriminatory. C) The vesting requirements for a SEP prohibit forfeitures. D) The SEP's trustee is subject to ERISA's prohibited transaction excise tax penalties.

Answer is C. SEP contributions must be 100% vested (i.e., nonforfeitable). SEPs consist of individual IRAs; there is no trustee for a SEP plan. The contribution formula of a SEP is not required to be fixed. Employer contributions to a SEP are not subject to payroll taxes.

Benjamin Scott, age 42, earns $110,000 a year and wants to establish a profit-sharing plan. He employs four people whose combined salaries are $58,000 and who range in age from 24 to 30. The average employment period for all employees is three-and-a-half years. Which vesting schedule is best suited for Benjamin's plan? A) Five-year cliff vesting B) Three-year cliff vesting C) Two-to-six-year graded vesting D) Three-to seven-year graded vesting

Answer is C. The choices for vesting in a defined contribution plan are: (1) 100% full and immediate, (2) three-year cliff, or (3) two-to-six graded. Because of the average length of employment, the most suitable vesting schedule from Benjamin's point of view (cash flow if termination occurs and reallocated forfeitures to Benjamin) is graded vesting. Five-year cliff vesting and three-to-seven-year graded vesting are not available to Benjamin's company for a profit-sharing plan. LO 3.1.

Which of the following is a retirement plan that isn't easily understood by employees, the employee assumes the investment risk, favors older plan participants, and does NOT permit elective deferrals? A) Money purchase pension plan B) Traditional defined benefit plan C) Target benefit pension plan D) Traditional profit-sharing plan

Answer is C. The question is describing a target benefit pension plan. LO LO 2.3.2

All of the following statements regarding a traditional profit-sharing plan are correct except A) profit-sharing plans are suitable for companies that have unstable cash flows. B) profit-sharing plans are qualified defined contribution plans. C) a company that adopts a profit-sharing plan must make contributions each year. D) company profits are not a prerequisite for employer contributions.

Answer is C. company that adopts a profit-sharing plan is not required to make contributions each year because plan contributions are flexible and employers are not required to make annual contributions. As a discretionary plan, profit-sharing plans are suitable for companies that have unstable cash flows. Company profits are not required. The company can make the contribution out of retained earnings if it chooses. LO 3.1.1

Which of the following statements is CORRECT in describing the integration of a 5% money purchase plan? A) The permitted disparity is 5.7%. B) The maximum excess contribution percentage is 15.7%. C) The offset integration method may be used. D) The maximum excess contribution percentage is 10.0%.

Answer is D. A money purchase plan with a base contribution percentage of 5% will have a 5% permitted disparity (lesser of the 5% base or 5.7%); therefore, the excess contribution percentage will be 10% (5% base + 5% permitted disparity).

Which of the following is CORRECT about contributions made to a cash balance pension plan? A) All contributions are made by the employee. B) Both the employer and the employee make contributions. C) Neither the employer nor the employee make contributions. Cash balance plans are funded entirely by payroll taxes. D) All contributions are made by the employer.

Answer is D. All contributions to the account-based cash balance plan come from the employer and must reflect a uniform allocation formula based on compensation. LO 2.2.1

The maximum service requirement that a thrift plan may impose as a condition of participation is A) one year. B) one-and-a-half years. C) six months. D) two years.

Answer is D. By law, the maximum service requirement that a thrift plan may impose is two years of service. An employee must become a participant in a thrift plan on the next plan entry date following the later of attainment of age 21 or the completion of two years of service. Of course, a more liberal participation requirement may be imposed. The maximum for a 401(k) is one year.

Which of the following describes a basic provision of a SIMPLE IRA? A) SIMPLE IRA plans can be arranged to allow for in-service loans for up to 50% of the account balance, but not to exceed $50,000. B) An employer may add a SIMPLE IRA plan to an existing defined benefit plan to allow employees to make elective deferrals. C) Only employers that average fewer than 20 employees can establish a SIMPLE IRA. D) A SIMPLE IRA must satisfy both the ADP and ACP nondiscrimination tests. E) One contribution formula an employer can use under a SIMPLE IRA is to make a 2% nonelective contribution on behalf of each eligible employee with at least $5,000 in current compensation.

Answer is E. SIMPLE IRA plans are available to employers with 100 or fewer employees and with no other qualified retirement plan. The employer contribution requirement may be satisfied by either a 3% matching contribution formula or a 2% nonelective contribution for each employee with current-year compensation of $5,000 or more.

Joe has worked for XYZ Co. for 30 years and is a participant in his employer's traditional defined benefit pension plan. He is retiring this year. The plan formula provides a pension equal to the average of the participant's final three years of compensation. Joe's final three years of compensation were $200,000, $230,000, and $290,000. What will be the amount of Joe's pension under the plan in 2020? A) $230,000 B) $238,333 C) $240,000 D) $221,000

Applying the 2020 covered compensation limit of $285,000, Joe's final three years of compensation averages to $238,333. However, the maximum defined benefit pension payable from a traditional defined benefit pension plan is $230,000 (2020).

Which of the following are minimum coverage tests for qualified retirement plans? Minimum allowed discrimination Average benefits percentage test Ratio test Maximum compensation test

Answers 2 and 3 The two minimum coverage tests for qualified retirement plans are the average benefits percentage test and the ratio test. To be qualified, a retirement plan must meet at least one of these tests if the plan does not meet the percentage (safe harbor) test.

Which of the following describe differences between a tax-advantaged retirement plan and a qualified plan? IRA-funded employer-sponsored tax-advantaged plans may not incorporate loan provisions. Employer stock distributions from a tax-advantaged plan do not benefit from NUA tax treatment.

Both I and II are correct. IRA-funded employer-sponsored tax-advantaged plans are SEPs, SARSEPs, and SIMPLE IRAs.

A type of qualified retirement plan in which participating employees are divided into groups or classes and each group or class receives an employer contribution equal to a percentage of compensation is A) a thrift plan. B) a new comparability plan. C) an age-based profit-sharing plan. D) a traditional profit-sharing plan.

In a new comparability plan, the participants are divided into groups or classes, and each group or class receives a contribution equal to a certain percentage of the contribution. LO LO 3.2.1

Gordon has met the two tests required for a hardship withdrawal from his profit-sharing plan with his employer. Which of the following is a reason for why money may be withdrawn using the hardship withdrawal rules? A) Larry, Gordon's cousin, has asked him for a loan to pay his college costs this semester. Larry is not a dependent of Gordon. B) Gordon, who has had financial reversals, is using the withdrawal to prevent a foreclosure on his vacation home. C) None of these are reasons to qualify for a hardship withdrawal. D) Gordon needs the money to pay for unreimbursed medical expenses for his spouse.

Only the cash needed for the unreimbursed medical expenses for Gordon's spouse qualifies for a hardship withdrawal. Money may only be withdrawn under the hardship withdrawal rules for the following reasons: Payment of unreimbursed medical expenses or for funeral costs Purchase of a primary residence Payment of higher education expenses for the participant, the participant's spouse, or dependent children Payment necessary to prevent foreclosure on the participant's primary residence If a hardship withdrawal is approved and made, the distribution is taxable and a possible 10% penalty applies. LO 3.1.1

Which of the following transactions between a qualified plan and a disqualified person are prohibited transactions, as defined by ERISA? The employer's purchase of a mortgage note in default from the plan for more than fair market value The sale of undeveloped land to a qualified plan for a bargain price The acquisition of employer stock by a profit-sharing plan for full and adequate consideration A loan to a 100% shareholder-participant in an amount proportionate to the amounts available to other plan participants, as specified in the plan documents

Options I and II answer, respectively, are prohibited transactions for the following reasons. The sale or exchange of property (i.e., a mortgage note in this situation) between a plan and a disqualified person results in a prohibited transaction. The sale of the undeveloped land is a prohibited transaction for the same reason. The federal courts have held that an in-kind contribution of employer-owned property to a plan in satisfaction of the employer's funding obligation is a prohibited transaction (i.e., a sale or exchange). Options III and IV are not prohibited transactions. ERISA specifically exempts the acquisition or sale of qualifying employer securities by an individual account plan (such as a profit-sharing plan) from prohibited transaction treatment. Loans in accordance with plan provisions are available to shareholder-participants who are disqualified persons.

The actual contribution percentage (ACP) test is required for which of the following plans? A) Plans that do not permit employees to make after-tax contributions B) Plans that allow employee elective deferral contributions C) Plans that provide employer-matching contributions D) Plans funded exclusively by participant elective deferrals

The ACP test is required for all plans that provide employer-matching contributions or employee after-tax contributions. The actual deferral percentage (ADP) test is applicable to plans that allow employee elective deferral contributions.

Max is the finance director for Bland Foods, Inc. He is trying to implement a new qualified retirement plan for the company. There are numerous federal guidelines with which the company must comply. Which of the following federal agencies is tasked with supervising the creation of new, qualified retirement plans? A) FINRA B) ERISA C) PBGC D) IRS

The Internal Revenue Service (IRS) carries out the task of supervising the creation of new, qualified retirement plans. LO 1.1.1

What are the requirements and effect of an eligible investment advice arrangement under the Pension Protection Act? An eligible investment advice arrangement allows a plan fiduciary to give advice including recommending their own proprietary funds without violating fiduciary rules. The investment advisor's fees must be neutral. An unbiased computer model certified by an independent expert to create a recommended portfolio for the client's consideration is used. Investment advisors for IRAs may only use the unbiased computer model option when providing eligible investment advice.

Statements I, II, and III are correct. Investment advisors for IRAs may only use the neutral fee option and not the computer model option when providing eligible investment advice.

Which of the following statements regarding the disadvantages to the employer of SEP plans is CORRECT? Employees cannot rely on a SEP plan alone to provide an adequate retirement benefit, which may hinder appreciation of the plan by employees. The employer bears the investment risk under the plan. If an employer maintains a SEP plan and a qualified plan, contributions to the SEP plan reduce the amount that may be deducted for contributions to the qualified plan. The special rule for calculating deductible contributions on behalf of an owner-participant in a qualified plan also applies to a SEP plan.

Statements I, III, and IV are disadvantages of a SEP plan to an employer. Statement II is false. The employee bears the investment risk under the plan.

Which of the following is NOT an example of a qualified retirement plan? A) New comparability plan B) Section 401(k) plan C) Section 403(b) plan D) Employee stock ownership plan (ESOP)

The answer is a Section 403(b) plan is a tax-advantaged plan but not an ERISA-qualified retirement plan. While tax-advantaged plans are very similar to qualified plans, there are some minor differences.

Assume a company's goal is to maximize retirement benefits to the highly compensated employees, who also happen to be the oldest employees. Which of the following best accomplishes this goal if the company is installing a new plan? A) A target benefit pension plan B) An age-weighted profit-sharing plan C) A defined benefit pension plan D) A money purchase pension plan

The defined benefit pension plan favors older participants and generally allows larger contributions than other plans. Age-based profit-sharing plans and target benefit pension plans also favor older participants. However, age-based profit-sharing plans and target benefit pension plans are both defined contribution plans that are subject to the annual additions limit (for 2020 a maximum contribution of no more than the lesser of 100% of an employee's compensation or $57,000).

Angela has received a sizable inheritance. She makes $12,000 a year working part time. She has been deferring 5% into her 401(k) at work even though there is no match. What is the maximum she can defer into the 401(k) in 2020? A) $12,000 B) $57,000 C) $6,000 D) $19,500

The maximum annual addition for 2020 is the lesser of 100% of compensation, or $57,000. Thus, Angela is limited to $12,000.

George is an executive with the MAH Company. For 2020, the maximum annual contribution under a money purchase pension plan on behalf of a participant is the lesser of 100% of the employee's covered compensation, or A) $57,000. B) $19,500. C) $225,000. D) $285,000.

The maximum annual contribution for a money purchase pension plan on behalf of a participant is subject to the annual additions limit, which is the lesser of 100% of the participant's covered compensation, or $57,000 (2020).

Which of the following is a defined benefit pension plan that promises a benefit based on a hypothetical account balance versus a traditional defined benefit pension plan, which promises a monthly retirement benefit for life? A) Target benefit pension plan B) Cross-tested plan C) Age-weighted plan D) Cash balance pension plan

The plan described is a cash balance pension plan. Age-weighted plans allocate contributions to participants in such a way that when contributions are converted to equivalent benefit accruals (stated as a percentage of compensation), each participant receives the same rate of benefit accrual. Cross-tested plans are defined contribution plans that test whether the contribution formula discriminates in favor of the highly compensated employee by converting contributions made for each participant into equivalent benefit accruals. Target benefit pension plans are hybrid retirement plans that use a benefit formula like that of a defined benefit pension plan and the individual accounts like a defined contribution plan. LO 2.2.1

What is the penalty for a prohibited transaction? A) 50% B) 15% C) 25% D) 10%

The prohibited transaction penalty is 15%.

RQZ Company employs 200 nonexcludible employees, 20 of whom are highly compensated. Sixteen of the 20 highly compensated and 125 of the non-highly compensated employees benefit from the RQZ qualified pension plan. The average benefits accrued for the highly compensated is 8%. The ratio test for the plan just listed is A) 86.8%. B) 69.4%. C) 80.0%. D) 70.0%.

The ratio test is 86.8%, calculated as follows: NHC coverage = 125 ÷ 180 = 69.44% HC coverage = 16 ÷ 20 = 80% 69.44% ÷ 80% = 86.8% (satisfies the ratio coverage test)

Which of the following plans is a qualified defined contribution profit-sharing plan that gives participants the option of reducing their currently taxable compensation by contributing on a pretax basis to an individual account for retirement purposes? A) Section 403(b) plan B) Section 401(k) plan C) Money purchase pension plan D) Simplified employee pension (SEP) plan

The statement describes a Section 401(k) plan. A Section 403(b) plan is a retirement plan that is available to certain tax-exempt organizations and to public schools that allows employee elective deferrals but is not a qualified plan. A money purchase pension plan is a defined contribution plan that specifies an employer level of contribution (for example, 10% of salary) to each participant's account each year. A SEP plan is not a qualified plan and does not allow employee elective deferrals. LO 3.1.1

Big Bucks Bank, as the plan trustee for the XYZ Corporation profit-sharing plan, has entered into a loan with the plan secured by the individual account balances of the plan participants. What has just occurred? A) A contribution to the plan consistent with the annual additions limit B) A disqualified loan C) A prohibited transaction D) A financial obligation incurred in the ordinary course of business

This is an example of a prohibited transaction under ERISA and the Internal Revenue Code. The lending of money between a qualified plan and a disqualified person, such as the plan trustee/bank here, is prohibited and subject to an initial 15% prohibited transaction penalty. If the transaction is not corrected, there is an additional 100% penalty for the amount of the prohibited transaction. Also, the party at interest is personally liable for the penalty if the plan assets are not sufficient to pay the penalty.

Total annual contributions to an individual participant in a traditional Section 401(k) plan are limited in 2020 to A) $19,500. B) the lesser of 100% of compensation, or $57,000. C) $285,000. D) $13,500.

Total annual contributions to a participant's account are limited in 2020 to the lesser of 100% of employee compensation, or $57,000, with only the first $285,000 of employee compensation considered in the contribution formula. The total contribution is made up of the worker contribution, the employer contribution, and reallocated forfeitures. The worker contribution alone is limited to $19,500 in 2020 for those 49 and younger.

In a profit-sharing plan that is integrated with Social Security and uses a 4% base contribution rate, what is the maximum excess contribution percentage that may be applied? A) 8.0% B) 5.5% C) 9.7% D) 5.7%

Under the permitted disparity rules, the maximum permitted excess contribution percentage is the lesser of two times the base percentage; and the base percentage plus 5.7%,, resulting in a total excess contribution percentage of 8.0% (2 × 4%).

Which of the following statements is CORRECT in describing the effects of actuarial methods or assumptions on contributions to a defined benefit pension plan? A) Using salary scales tends to reduce the cost for funding younger employees' benefits. B) The higher the turnover rate assumed by the actuary, the lower the required employer contribution for the year. C) The lower the projected investment earnings on plan assets over the life of the plan, the lower the required employer contribution for the year. D) The higher the interest rate assumed by the actuary, the higher the required employer contribution for the year.

answer b. Because defined benefit plans must apply forfeitures to reduce the employer contribution, an actuarial assumption of high turnover in a plan year will result in a lower required employer contribution.

Which of the following may be taxable to a profit-sharing plan participant even when there is no distribution from the plan? A) Investment earnings B) Life insurance premiums for a policy held by the plan C) Employer contributions D) Employee contributions

answer is bWhen life insurance is provided for a plan participant through a profit-sharing plan, the economic value of the pure life insurance cost or premium is taxed to the participant using either Table 2001 or the insurance company's actual term rates. This amount is offset by any employee contribution to the plan. LO 3.1.1


Related study sets

Adult Health-Test 3-Immunity-Rheumatoid Arthritis (Exemplar 8.3) & Systemic Lupus Erythematosus (Exemplar 8.4)

View Set

ANTHROPOLOGY TEST 2 GUIDE - Chapter 8: Kinship & Family

View Set